Söker efter växthuseffekt

 

56 frågor/svar hittade

Universum-Solen-Planeterna [252]

Fråga:
Kommer inlandsisen på nord- och sydpolen någonsin att smälta?
/

Svar:
Det tror jag inte. Så vitt jag vet har det alltid funnits is vid polerna. Det kan hända att om bilar och industri släpper ut för mycket koldioxid i luften så kommer växthuseffekten att öka och temperaturen att stiga mycket så att isen kan smälta. Det är väldigt svårt att fundera ut exakt vad som ska hända och det finns många olika åsikter om hur det blir i framtiden.

*

Blandat [391]

Fråga:
1. Har ozonlagret något samband med växthuseffekten?

2. Hur påverkar CFC-gaserna (freon) växthuseffekten och ozonlagret?

3. Man säger ju att klimaten blir mycket varmare av växthuseffekten, varför har då medeltemperaturen bara ökat med en halv grad på 100 år?
/

Svar:
1 Egentligen inte. Ozonlagret stoppar den ultravioletta strålningen. Växthuseffekten påverkas främst av koldioxidhalten i atmosfären. Om ozon-lagret försvinner kan den ultravioletta strålningen påverka växtligheten på jorden så att koldioxidhalten förändras och därmed förändras växthuseffekten.

2 Freonet förstör ozonlagret. Nobelpriset i kemi 1995 utdelades till de forskare som klargjorde sambanden mellan freon och ozonlagret. Du kan läsa mer om detta i vetenskapsakademiens beskrivning (länk 1 nedan). Freonet är dessutom en växthusgas.

3 Först en förklaring. Växthuseffekten har alltid funnits och är väldigt viktig för jordens klimat. Utan denna effekt skulle det både vara kallare och större skillnad mellan dag- och nattemperatur. Vad som händer är att denna effekt förstärks något om halten koldioxid ökar i atmosfären.

Vi vet inte säkert vad som händer när växthuseffekten förstärks. Det kan hända att temperaturen stiger. Det kan också hända att vi får fler moln som reflekterar solstrålningen så att temperaturen inte stiger.

1 http://nobelprize.org/chemistry/laureates/1995/press.html

*

Blandat, Universum-Solen-Planeterna [830]

Fråga:
Varför uppkommer istider?
/Veckans fråga

Ursprunglig fråga:
1) Kommer vi få en ny istid?

2) Vad beror det på i så fall?

3) Varför blir det klimatförändringar?

4) Vad är det som påverkar klimatet?

5) Hur kommer klimatet se ut om 100 år?

6) Går det att reparera ozonlagret?
/Anna K, Borgarskolan, Malmö

Svar:
Det är mycket svåra frågor du ställer Anna! Jag skall försöka svara så gott det går. Först: Vad menar vi med istid? Det är när en stor del av t.ex. norra Europa, Asien och Nordamerika är täckt av inlandsis.

Figuren nedan från Wikimedia Commons visar ändring i jordens temperatur (blå kurva), halten CO2 (grön kurva) och mängd damm (röd kurva). Under de senaste 400000 åren har vi alltså haft 4 perioder när klimatet varit 10 grader kallare än vad temperaturen är nu. Dessa kalla perioder kallas istider och kontinenterna har alltså delvis varit täckta av ett istäcke på flera km:s tjocklek.

Man ser att temperaturen och halten CO2 följer varandra mycket nära. Anledningen är att kallt vatten kan lösa mer CO2 än varmt vatten. Alltså: Kallt klimat - mer koldioxid i haven och mindre i atmosfären. Vid varmt klimat är det tvärtom.

Observera orsak och verkan här. Det är inte den högre halten av CO2 som är orsak till att det är varmare nu än under istiden. CO2 är visserligen en s.k. växhusgas - se nedan - men en fördubbling av nuvarande halt skulle bara ändra jordens medeltemperatur med en grad och skillnaden i temperatur mellan istid och icke-istid är c:a 10 grader. Det är alltså temperaturändringen som orsakar förändringen i CO2-halt, inte tvärt om!

Resonemanget i ett dokument från Columbia University (numera försvunnet) är alltså helt felaktigt:

Past relationships between atmospheric CO2 and surface temperature

We can learn about earth climate sensitivity to past variations in atmospheric CO2 by drilling into ice sheets. Ice sheets record past concentrations of atmospheric CO2 by trapping bubbles of ancient air as the ice sheet forms. The figure below shows the relationship between CO2 in the atmosphere and surface temperatures over Antarctica spanning the last 150,000 years.

As you can see, there is a very close relationship between surface temperatures and atmsopheric CO2 levels. Note, however, the present mismatch between the current high levels of CO2 (around 365 ppm) and the relatively unchanged surface temperatures. If past history is a guide to the future, the data in this plot suggest we are due for very significant global warming.

Figuren man refererar till är i princip nedanstående. Man har, som synes, problem att förklara nuvarande temperatur med en CO2 halt av över 370 ppm. Förklaringen ges ovan: det är temperaturändringen som orsakar ändringen i CO2 halt, inte tvärt om!

Nu till dina frågor.

1) Ganska säkert, men vi vet inte exakt när. De senaste 4 istiderna visas på figuren nedan. Man ser att de återkommer med stor regelbundenhet med en period på c:a 110-120 tusen år. Interglacialperioderna (varma perioder mellan istiderna) är emellertid ganska korta, så om c:a 10000 år blir det nog en ny istid om det inte händer något annat innan dess.

2) Man tror att istiderna beror på att jordens bana påverkas av jätteplaneterna Jupiter och Saturnus. Det är flera aspekter på jordens rörelse som påverkas, men viktigast är antagligen att excentriciteten (avlångheten) hos jordbanan ändras periodiskt (se Milankovitch cykler ). Medelavståndet till solen ändras inte, men enligt Keplers andra lag (se Keplers lagar ) kommer jorden att tillbringa längre tid på stort avstånd från solen eftersom banrörelsen är långsammare där. En excentrisk bana ger alltså lägre temperatur, en cirkulär högre temperatur. Påverkan av övriga variationer beskrivs i fråga 14214 .

3,4) Jordens temperatur beror på balansen mellan instrålning (ljus från solen) och utstrålning (värmestrålning från jorden), se strålning, in-/ut- . Solens utstrålning är mycket stabil - åtminstone på tidsskalor på några 100 miljoner år. Instrålningen till jorden kan alltså endast ändras genom ändringar i jordens rörelse (se ovan) eller att jordens reflektionsförmåga (sk albedo) ändras. En del av det inkommande ljuset reflekteras direkt och försvinner ut i rymden. Ökning i albedo kan t.ex. åstadkommas av mer moln eller mer is/snö. Moln och is/snö är ju vitt, dvs har hög reflektionsförmåga. De orsakar därmed minskning i nettoinstrålningen och därmed en sänkning i temperaturen.

Om golfströmmen skulle upphöra, så får vi säkert åtminstone en liten istid. Det kan vara så att det bara är slumpen som orsakar istider: man kan tänka sig att även förekomsten av inlandsis över norra Europa kan vara ett stabilt system även utan en ändring i instrålningen.

Även om instrålningen är konstant kan man ändra balansen genom förhindra utstrålningen. Vissa gaser, t.ex. vattenånga och koldioxid absorberar värmestrålningen från jorden och hindrar den därmed att försvinna ut i rymden. Detta medför en uppvärmning av jorden, se växthuseffekten .

5) Ingen - inte ens Polman - vet.

6) Inte med nuvarande teknik. Om vi slutar förstöra ozonlagret, så kommer det med tiden att reparera sig själv.

Se vidare Ice_age .



/Peter E

Nyckelord: istider [8]; *geologi [16]; växthuseffekten [36];

1 http://www.planetseed.com/relatedarticle/sun-and-earth-and-temperature-change

*

Energi [908]

Fråga:
Vad är positivt med kolkraft?
/Nina J, Komvux, Enköping

Svar:
Inte mycket! Det är smutsigt och ger upphov till koldioxid som kan påverka jordens klimat (växthuseffekten). Det enda jag kan komma på som är positivt, är att det är svårt att föreställa sig riktigt stora katastrofer med kolkraft (gruvolyckor vid brytningen kan naturligtvis förekomma). Med kärnkraft och med vattenkraft kan man tänka sig ytterst osannolika scenarion där även allmänheten kan drabbas hårt.
/Peter Ekström

*

Energi, Kraft-Rörelse [1022]

Fråga:
Skulle man kunna använda sig av tyngdkraften på något sätt som energikälla?
/Veckans fråga

Ursprunglig fråga:
Jag undrar om man skulle kunna använda sig av tyngdkraften på något sätt som energikälla.
/Ludvig H, De la Gardie, Lidköping

Svar:
Om man släpper ett föremål i ett tyngdkraftfält, så accelereras det och får en viss hastighet som beror av fallhöjden, h. Denna rörelseenergi kan användas som energikälla. Problemet är bara att när föremålen på höjden h är slut, så är energikällan uttömd.

Det finns emellertid ett "föremål" som används på detta sätt som energikälla, nämligen vattnet i sjöar och älvar. Solens värme förångar vattnet från haven, moln bildas och regnet faller i högt liggande områden. Hälften av Sveriges elenergi kommer från vattenkraft. Indirekt är alltså vattenkraft en sorts solenergi.

Vattenkraften är ganska skonsamt mot miljön. Det ger inga utsläpp och bidrar därför inte till växthuseffekten . Utbyggnad av älvar förstör emellertid den lokala naturen och fisket genom att dammar översvämmar stora områden och att älven och vattenfallen försvinner.


Trollhättans vattenkraftverk (från Vattenfalls webbtjänst, se länk nedan).

Länk: Vattenfall - Energikunskap
/Peter Ekström

Nyckelord: vattenkraft [7];

*

Blandat [1076]

Fråga:
Säg att växthuseffekten fortskrider och de stora isarna smälter och "lägger jorden i blöt". Vilka landområden kommer då fortfarande sticka upp ur vattnet? Om bara Mount Everest gjorde det, skulle en människa då kunna andas normalt där? Alltså, ändras trycket om berget skulle "kortas av" underifrån?
/Nina !, Norreportskolan, Ystad

Svar:
Man beräknar att vattenytan skulle stiga med ungefär 70 meter. De isar som redan flyter på vattnet (främst i Arktis) bidrar inte alls till havsytans höjning om de smälter.

Försök Fyll ett glas med vatten och några isbitar precis upp till kanten. Rinner något vatten ut när isen smälter?

Däremot kommer den is som befinner sig på land (främst i Antarktis) att bidra till vattenhöjningen. Om vattentemperaturen stiger i havet så kommer dessutom vattnet att utvidga sig, få lägre densitet, vilket också bidrar till havsytans höjning.

Vattnet kan alltså inte stiga "så att bara Mount Everest är kvar" men låt oss ändå leka med den tanken. Då trycks även atmosfären uppåt och lufttrycket vid havsytan ändras inte när havsytan stiger eftersom det är luftens tyngd som ger trycket.

Undersök Titta på en karta och se efter vilka områden av Sverige som skulle komma under vatten om havsytan steg med 10 meter.


/GO, KS

*

Värme [1547]

Fråga:
Hur lång tid kommer det att ta innan isarna på polerna smälter bort?
/Cecilia O, Fuxerna, Lilla Edet

Svar:
Du tänker förmodligen på växthuseffekten. Det går nog inte att ge ett exakt svar på din fråga.

Däremot vet vi, att klimatet kan skifta snabbt, kanske på 10 år. Det som driver havsströmmarna i Atlanten, är att kallt vatten sjunker i trakten av Grönland. Tack vare detta har vi Golfströmmen, som ger oss behaglig temperatur på vintern. Skulle denna cirkulation upphöra, fick vi en drastisk klimatförändring.
/KS

*

Energi [1782]

Fråga:
Vad är fossila bränslen?
/Veckans fråga

Ursprunglig fråga:
Jag skulle vilja veta lite om fossila bränslen. Vad är fossila bränslen? Hur de används? För- och nackdelar?
/malena h, Eksjö gymnasium, Ekjsö

Svar:
Exempel på fossila bränslen: kol, olja, naturgas. De kallas fossila därför att de finns i gamla geologiska formationer, och skulle ha blivit kvar där, om inte vi hade plockat upp dom för att elda med (naturgas borde alltså i konsekvensens namn kallas fossilgas). Problemet är att vi därmed ökar koldioxidhalten i atmosfären. Detta kan ge upphov till en temperaturhöjning med svåröverskådliga följder, den så kallade växthuseffekten . Fördelen är att de är relativt lättillgängliga, lättanvända och billiga.

Motsatsen kallas fönybara bränslen, som trä, halm, biogas, etanol, rapsolja. De gröna växterna tar ju upp koldioxid ur luften. När vi eldar trä frigörs visserligen kodioxid, men det blir inget nettotillskott eftersom trädet har tagit upp lika mycket när det växte. Se fråga 1129 för det mer generella begreppet förnybara energikällor.

Wikipedia säger om bildandet av fossila bränslen:

Enligt den biogeniska teorin har jordens petroleumtillgångar bildats då förhistoriska alger och plankton lagt sig på sjöars och havs botten under syrefria förhållanden. Detta organiska material har begravts under stora lager sediment. Genom högt tryck och hög temperatur har det omvandlats kemiskt, först till kerogen och sedan genom ytterligare tryck och värme till kolväten i gas- eller vätskeform (det vill säga naturgas och petroleum).

Växter på land bildar dock främst kol. Stora delar av jordens kolreserver härstammar från den geologiska perioden karbon.

Kol kommer alltså antagligen från jättestora ormbunksliknande träd, se den fantasifulla illustrationen nedan från Wikimedia Commons.

Alla fossila bränslen kommer alltså från växter/alger som med hjälp av solljus omvandlar koldioxid och vatten till kolhydrater:

CO2 + H2O + energi (solljus) --> CH2O + O2

CH2O är en förenklad formel för ett kolhydrat. Under lång tid och vid högt tryck och hög temperatur omvandlas sedan kolhydraterna till kolväten (olja, fossilgas) eller kol. Energin i dessa bränslen är alltså inget annat än lagrat solljus!

Se vidare länk 1, Fossila_bränslen och Fossil_fuel .

Fundera: Hur är det med torv? Är torv fossilt eller förnyelsebart?

Tillägg 13/12/2013:

Ovanstående beskrivning av bildandet av fossila bränslen är mycket väl etablerat i vetenskapen. Det finns emellertid en liten minoritet som tror på ett icke-biologiskt scenario.

Det mest direkta argumentet för biologiskt ursprung är att 13C/12C-förhållandet i organiskt material är mindre än i oorganiskt kol. Fossila bränslen har detta lägre förhållande, se fråga 19264 .

Här är ett utdrag av Bradley J Dibble (länk 2) som argumenterar för den traditionella synen med argumentet att fossila bränslen innehåller fossila biomarkörer (identifierbara rester av växter/djur).

The abiotic theory is controversial and has a number of flaws, however. For one, it doesn’t predict deposits of oil as well as the biogenic theory does. Oil deposits are typically found close to fault lines because that’s where two tectonic plates meet, and ocean sediments can be more easily buried in those regions. Also, oil deposits usually have biomarkers, little telltale signs of life. For the abiotic theory to work, those markers have to be explained somehow; it fills in that hole by suggesting microbes must have been feeding on the petroleum. The biogenic theory easily explains why such evidence of life would be present, however, given that they originated from the remains of once-living plants.


/KS/lpe

Nyckelord: energikällor [26]; fossila bränslen [13]; *geologi [16]; växthuseffekten [36];

1 http://www.energyquest.ca.gov/story/chapter08.html
2 http://bradleydibble.authorsxpress.com/2012/04/29/the-origin-of-coal-oil-and-natural-gas-fossil-fuels-or-something-else/

*

Blandat [3613]

Fråga:
Hej, jag och min kompis har några frågor: 1)Vad består ozonskiktet av? 2)Det är ju ett hål ovanför antarktis, hur upptäcktes det? 3)Vad kommer att hända i framtiden, kommer hålet att försvinna eller bli större? 4)Kommer hålet förvärra växthuseffekten eller "skydda" mot den? 5)Vi har hört att det finns ställen där det är väldigt tunt skikt, kommer det att bli "hål" där eller kommer det att bli "tjockare"? Tack på förhand för hjälpen! /Liza & Niina
/Liza J, Tegelviken, Eskilstuna

Svar:
Ordet ozonskiktet är en smula vilseledande. Det är i sjäva verket ett område i jordatmosfären, där en liten del av syret förekommer som ozon (O3) i stället för vanligt syre (O2). Det är på en höjd av ungefär 15 - 50 km. Ozonet där har stor betydelse för livet på jorden, eftersom det absorberar (tar bort) skadligt ultraviolett ljus från solstrålningen. Det finns en utmärkt artikel om detta i Nationalencyklopedin . Läs den och återkom gärna om det är något ni vill diskutera.

Sök på ozon i denna databas. Här finns mycket att hämta. 
/KS

*

Blandat [4035]

Fråga:
Kommenter till frågor och svar om Bermuda-triangeln. (ID 3999 och 4002) Detta är en fråga som intresserat mig lite och jag såg en gång ett program på Vetenskapens Värld om just detta med gas. Enligt det programmet så skulle gas-massorna strömma upp till ytan vid smärre jordskalv. Då bubblar vattnet (förklaring till myten om kokande vatten), om gasen antänds kan det se ut som om vattnet brinner. Båtar sjunker, (fenomet finns hos oljeborrtorn som stött på en gasficka) och flygplan skulle kunna sprängas om gasen antänds av deras motorer. Den lösa segmenterade botten skulle sedan täcka över vrakdelarna så att det "spårlöst försvinner". Det är väl en teori, men jag tyckte det lät rätt vettigt... Vad tycker ni? /Adrian PS. Sen finns det ju självklart mycket skrock runt Bermuda-triangelns olyckor.
/Adrian S, Erik Dahlbergsgymnasiet, Jönköping

Svar:
Jo det ligger väl något i dina ideer. Men den största faran med metanisen (se länk nedan), som enligt den här teorin smälter och avger gasen, är att växthuseffekten skulle orsaka en kraftig uppvärmning av världshaven. Det skulle kunna leda till en katastrofal förgasning av metanisen. Metan är i sig en effektiv växthusgas, och växthuseffekten skulle förstärkas kraftigt. Det finns tecken på att just detta hände för 55 miljoner år sedan. Jordens medeltemperatur steg 5 - 7 oC. Många arter dog ut, men denna klimatkatastrof gynnade tydligen däggdjuren. Det är först efter denna händelse som däggdjuren uppträder på allvar.
/KS

Se även fråga 3999 och fråga 4002

1 http://www.llnl.gov/str/Durham.html

*

Universum-Solen-Planeterna [5467]

Fråga:
Varför finns det bara liv på våran planet??
/petra n, strömsnässkolan, strömsnäsbruk

Svar:
Klimatet har gått åt helt olika håll för våra närmsta grannplaneter. Venus har råkat ut för extrem växthuseffekt, så att temperaturen på ytan är nästan 500 grader. Vatten bör ha funnits där, men det är borta nu. Mars har blivit en arktisk öken, med en typisk temperatur på ungefär -80 grader. Där har funnits flytande vatten, men allt har försvunnit ut i rymden eller frusit. Det är troligt att klimatet varit bättre förr på båda planeterna, och vi kan inte utesluta att där funnits liv. Det är helt uteslutet att liv, som vi känner det, skulle kunna finnas nu på Venus. På Mars kan det tänkas existera livsformer som lever djupt nere i marken, utan behov av solljus. Sådana har man hittat här på jorden. Länk 1 är ny (2012) forskning om förekomsten av flytande vatten på Mars.

En annan himlakropp som är intressant i detta sammanhang är Jupiters måne Europa. Den är helt istäckt, men man är ganska säker på att det finns en flytande ocean under. Vatten kan förekomma på Europa trots den mycket låga temperaturen eftersom Europa "knådas" av tidvatteneffekter från den massiva Jupiter, se fråga 2571 .
/KS/lpe

Nyckelord: liv i universum [9];

1 http://forskning.se/nyheterfakta/nyheter/pressmeddelanden/nystudiepekarpaokadeforutsattningarforlivpamars.html

*

Värme [6106]

Fråga:
Hur påverkar aerosolpartiklar i atmosfären solstrålningen?
/Anna J, Katedralskolan, Skara

Svar:
Det här är en komplicerad fråga. Man talar ju om växthuseffekten, orsakad främst av koldioxid från förbränning av fossila bränslen. Men denna producerar ju också aerosolpartiklar (små fasta partiklar) som hindrar solstrålningen att nå jordytan. Detta verkar åt motsatt håll. Hur dessa effekter balanserar varandra, är inte helt klart, men det arbetas mycket på det.
/KS

*

Energi [6297]

Fråga:
Hej! jag undrar vad skillnaden är när det gäller koldioxidutsläpp om man jämför fossilabränslen med biobaseradebränslen...snälla svara så fort som möjligt TACK!!
/Emma A, Sturegymnasium, Halmstad

Svar:
Problemet med fossila bränslen är att när man bränner dem ökar koldioxiden i luften (växthuseffekten). Det finns visserligen naturliga mekanismer för att återställa koldioxidhalten, men det tar miljontals år. Människan har mera bråttom än så.

För att producera biobränslen med fotosyntes tas koldioxid från luften. När man sedan bränner det, kommer samma mängd koldioxid ut igen. Koldioxidhalten blir i princip konstant. Fullt så här enkelt är det inte, men det är i huvudsak rätt.
/KS

Se även fråga 1782 och fråga 391

*

Blandat [7451]

Fråga:
Jag undrar vad det finns för globala tillämpningar med ir-värmestrålning?

Tack på för hand
/calle k, alströmer, alingsås

Svar:
Man kommer väl närmast att tänka på växthuseffekten. Vissa gaser (koldioxid, metan,vattenånga) är genomskinliga i synligt ljus, där den mesta solenergin kommer, men absorberar i infrarött, där jorden strålar ut värme. Ökar mängden av dessa gaser ökar, stiger temperaturen på jorden. Det har två konsekvenser. Havsvattnet värms, så att det expanderar. De stora inlandsisarna smälter. Detta gör att havsnivån stiger. Skulle all is smälta, skulle havsnivån stiga med 60 m. Det skulle få oerhörda konsekvenser för jordens befolkning. Den gas som det är mest tal om nu är koldioxid, som bildas vid förbränning av fossila bränslen (kol och olja).

Slå på Jorden i Nationalencyklopedin . Där finns en karta över hur Europa skulle se ut med 60 m högre havsnivå.
/KS

Se även fråga 6106 och fråga 6297

*

Värme [7474]

Fråga:
Hej! Vi håller på med ett arbete om växthuseffekten. Och det jag undrar om är: Varför förlorar solstrålarna energi när de studsar mot jorden, och inte förlorar energi när det studsar mot partiklar i ozonskiktet?? Tacksam för svar snabbt. MVH Jessica Qwarnström
/Jessica Q, Forssaklackskolan, Borlänge

Svar:
Ozonskiktet är knappast av betydelse för växthuseffekten. Växthusgaserna är framför allt koldioxid, metan och vattenånga. Kolla tidigare svar!
/KS

Se även fråga 7451 och fråga 391

*

Värme [10631]

Fråga:
Vatten har ju högst densitet vid +4. Hur är det med saltvatten? Vad är det för temp i botten på havet? Kristina
/Kristina B, Rudbeck, Örebro

Svar:
Ju högre salthalt desto lägre är temperaturen för den högsta deniseten. Slå på Hav i Nationalencyklopedin , där finns ett diagram över detta. Vid salthalten 2.47 % är vattnet tyngst precis vid fryspunkten, -1.33 oC. Normal salthalt i världshaven är 3.5 %, och det vattnet fryser vid -2 oC. När havsis fryser är det alltså -2 oC uppifrån och ända ner till botten. Detta sker mellan Island och Grönland, och detta åstadkommer en nedåtgående havsström, som drar söderut. Jordrotationen gör att den avviker västerut. Det vatten som på detta sätt avlägsnas från polarregionen ersätts av en ytlig havsström mot nordost (Golfströmmen). Utan den hade det varit ungefär 10 oC kallare här.

Inte bara Golfströmmen påverkas av det kalla "vattenfallet" utanför Grönland. Det har betydelse för strömmarna i hela värdshavet. Somliga fruktar att växthuseffekten hotar detta system. Om det stoppas blir det drastiska klimatförändringar runt om i världen.
/KS

*

Universum-Solen-Planeterna [10658]

Fråga:
Var kommer solens energi ifrån?
/Veckans fråga

Ursprunglig fråga:
Jag undrar hur kan det komma sig att solen har funnits i miljarder år utan att dess strålning varken har minskat eller ökat? dvs nästan allt behöver energi att fungera eller har en vis funktion som sedan brukar lägga av efter några år och det behövs en arbetsfunktion för att den ska fungera men solen det är ingen som driver den eller har skapat den hur kan den då fungera i miljarder år utan någons påverkan?
/fatma z, Apelgårdskolan, Malmö

Svar:
Solen befinner sig nu i ett skede med långsam utveckling (huvudserien). Då sker energiproduktionen genom "förbränning" av väte till helium i solens centrum. Detta har pågått i 5 miljarder år och kommer fortsätta i 5 miljarder år till. Därefter sker en rad drastiska förändringar innan solen slutar som en vit dvärgstjärna. För detta se nedan.

"Förbränningen" av väte till helium är inte vanlig kemisk förbränning som förbränning av kol till koldioxid genom tillsats av syre. Kemisk förbränning ger bara energimängder på ungefär eV (en mycket liten energienhet). Med en sådan förbränning skulle solen bara kunna lysa några tusen år.

En annan möjlig källa till solens energiutveckling som man funderade på i slutet av 1800-talet är gravitationsenergi. Solen skulle kunna frigöra energi genom att dra sig samman. Inte heller denna källa räcker till för att förklara energiutvecklingen under flera miljarder år.

För att förklara solens energiutveckling måste man ta till kärnfysik. Denna kunskap utvecklades under de första åren av 1900-talet. Den reaktion som ger solen energi sker vid c:a 15 miljoner grader i solens centrum och är mycket förenklat (i själva verket går reaktionen i flera steg, se bilden nedan och länk 1, Energiproduktion för detaljer):

4 1H --> 4He + energi

En väteatom har massan 1.007825032 massenheter (u) och en heliumatom har massan 4.002603250 u. Fyra väteatomer väger då 4.031300128 u. Skillnaden 0.028696878 u motsvarar en energi på 0.028696878*931.5 MeV = 26.7 MeV, dvs 26700000 eV, allså en miljon gånger mer än vad vanlig förbränning ger. Det är alltså denna stora förvandling av massa till energi (E=mc2) som är solens energikälla. Den relativa energiutvecklingen blir 0.028696878/4.031300128 = 0.71%, dvs 0.71% av massan väte omvandlas till energi.

Energin som frigörs i solens centrum transporteras till solytan med konvektion och strålning. Temperaturen vid solytan är c:a 6000 grader och vid den temperaturen sänds det ut temperaturstrålning med maximum intensitet i synligt ljus, se fråga 12409 .

En stjärna som är lite tyngre än solen kan mot slutet av sin utveckling även börja förbränna helium till kol:

3 4He --> 12C + Q

där Q är den utvecklade energin per reaktion:

Q = 3*m(4He) - m(12C) = 3*4.002603 - 12.000000 = 0.007809 massenheter = 0.007809*931.5 MeV = 7.274 MeV

Den relativa energiutvecklingen blir 0.007809/12 = 0.065%, dvs 0.065% av massan helium omvandlas till energi.

Att vätet räcker i c:a 5 miljarder år till betyder inte att livet kan finnas så länge. Allteftersom heliumhalten i solens centrum ökar, ökar också solstrålningen. Om 1 miljard är den 10 % högre än i dag. Det låter kanske inte så mycket, men det kommer leda till en skenande växthuseffekt i jordatmosfären, där sluttillståndet liknar förhållandet på vår grannplanet Venus. Där är temperaturen på ytan 450 oC, atmosfären består av kolsyra med ett tryck av 100 atmosfärer och molnen består av svavelsyra. Inget vatten finns nu på Venus. Under sådana förhållanden kan inget liv finnas.

Det är alltså naturlagarna som är så funtade att en stjärna av solens typ har en mycket lång period av långsam utveckling. Om utvecklingen hade varit mycket snabbare hade knappast liv hunnit utvecklas. Vi har idag mycket god kunskap om vad som sker i en stjärna som solen och hur den utvecklas.

Om du vill ha mer information, kolla sajten What will happen to the Solar System in the future . Den är på engelska. Det finns också en artikel oktobernumret 2002 av Sky and Telescope. Den är också på engelska.



/KS/lpe

Se även fråga 13731

Nyckelord: solens utveckling [4]; solens energiproduktion [9];

1 http://kasper.pixe.lth.se/NuclearPhysics/slideShow/f1/

*

Universum-Solen-Planeterna [10855]

Fråga:
Hej Jag undrar var jag kan hitta information om hur vädret har och kommer att förändras? Vilka teorier finns det mm Tacksam för svar mvh Sofie
/Sofie P, Stagneliusskolan, Kalmar

Svar:
En del information om detta kan du hitta i denna databas. Sök på växthuseffekten. Solens strålning ökar långsamt på grund av inre förändringar som ytterst orsakas av höjd heliumhalt. Någon gång långt i framtiden (kanske om 1000 miljoner år) kommer troligen jorden råka ut för en skenande växthuseffekt med ett sluttillstånd som liknar förhållandena på Venus. Atmosfär av nästan ren koldioxid med 100 atmosfärers tryck och en temperatur på 450 oC vid ytan. Moln av svavelsyra och inget vatten.
/KS

Nyckelord: växthuseffekten [36]; Venus [11];

*

Blandat [12168]

Fråga:
Varför uppstår surt regn?
/Veckans fråga

Ursprunglig fråga:
SURT REGN

Varför uppstår surt regn? Vad innehåller det? Hur kan man stoppa det? Var är det som mest?
/Michael E, Kirsebergsskolan, Malmö

Svar:
Surt regn uppstår när atmosfären innehåller framför allt svaveldioxid SO2 och NOx. Svaveldioxiden kommer från förbränning av kol och olja, eftersom kol och olja oftast innehåller en hel del svavel som förorening. Kväveoxiderna kommer från förbränning vid hög temeratur (luften innehåller ju syre och kväve).

Man kan stoppa det genom att sluta förbränna fossila bränslen (kol, olja) och använda bättre energikällor som inte ger upphov till utsläpp. Det finns mest svaveldioxid i områden där man har mycket industrier, t.ex. vissa delar av centraleuropa.

Naturgas (som egentligen borde kallas fossilgas) är bättre vad gäller svavel, men även den ger upphov till kväveoxider och koldioxid CO2, det senare ökar växthuseffekten. Denna är ett av mänsklighetens allvarligaste problem för närvarande, eftersom en höjning av jordens medeltemperatur på bara några grader skulle få allvarliga konsekvenser.

Se vidare Surt regn och Växthuseffekten .
/Peter E

Nyckelord: växthuseffekten [36];

*

Energi [12629]

Fråga:
Vilka fördelar och nackdelar har kraftverk som eldar med kol, olja och gas?
/Veckans fråga

Ursprunglig fråga:
1. Vilka fördelar och nackdelar har energikraftverk som eldar med kol, olja och gas?

2. Vad används energin från olja, gas och kol till?
/Anna S, Tullbroskolan, Falkenberg

Svar:
Här är två bra och lättillgängliga källor till information om energi: Vattenfall - Energikunskap och Svensk Energi - skolmaterial . Den senare innehåller, förutom information om olika energislag, fria overheadbilder och bilder för presentationer.

Nationalencyklopedin är också en bra informationskälla.

Alla energikällor utom vattenkraft och vind/vågkraft fungerar på så sätt att man värmer upp ett medium (vanligen vatten) till hög temperatur. Med hjälp av en turbin (som förvandlar värmeenergin till mekaniskt arbete) och en generator får man elektricitet. Problemet är att bara en liten del av den urspungliga energin (typiskt 30%) blir mekanisk energi. Resten är värme som ofta bara går ut i naturen. Ibland kan man utnyttja energin i kylvattnet för uppvärmning, och då blir effektiviteten lite bättre.

Eftersom alla är fossila bränslen - man bör egentligen kalla naturgas för fossilgas - så bidrar de till växthuseffekten, se Växthuseffekten . Olja har fördelen att vara lätt att förvara och förbränna, så det används som bränsle i de flesta typer av transportmedel.

Förutom utsläpp av koldioxid och en del svavel orsakar kol flera tusen döda gruvarbetare vid brytningen. Kinesiska gruvor har när det gäller detta speciellt dåligt rykte.

När man jämför olika energikällor är det viktigt att man inte bara konstaterar att källan x är skadlig, så vi skall inte använda den. Man måste även ta med i beräkningen hur skadliga är de alternativ som finns?

Bilden nedan är från 'Svensk energi i skolan'.



/Peter E

Nyckelord: energikällor [26]; fossila bränslen [13];

*

Blandat [12668]

Fråga:
Om växthuseffekten
/Veckans fråga

Ursprunglig fråga:
Hej. Fin sida ni har! Jag håller på att skriva ett projektarbete om hur växthuseffekten påverkar oss..Skulle behöva lite svar på frågor...

1 Är det människan som är orsaken till den globala värmehöjningen?

2 om ja. Finns det bevis för detta.

3 om nej vad/vilka är orsaken till höjningen då?
/lotta f, strömstad gymnasium, strömstad

Svar:
Bästa Lotta Fysikstuderande!

Växthuseffekten är den uppvärmning av jordytan som åstadkoms av jordens atmosfär. Effekten beror på att en del av den värme som strålar ut från jordytan värmer upp luften i atmosfären i stället för att stråla ut i rymden. Jorden blir därigenom varmare än den skulle ha varit om den hade saknat atmosfär. Växthuseffekten

Växhuseffekten orsakas av att synligt ljus från solen går nästan obehindrat igenom atmosfären och värmer upp jordytan. Eftersom jordytan är 15 grader C i medeltal strålar den i infrarött. Den infraröda strålningen absorberas till stor del av atmosfären i stället för att slippa ut i rymden, så vi får en uppvärmning. Figuren nedan från Greenhouse_effect illustrerar detta. De nedre panelerna visar att transmissionen av infraröd strålning är nära noll utom för några smala fönster.

Växthuseffekten är bra så länge den är lagom - utan växhuseffekten skulle jordens medeltemperatur vara -20 grader C! Det betyder att människan knappast funnits på jorden utan växthuseffekten.

Kvävet och syret i luften bidrar inte till växthuseffekten, det är framfår allt vattenånga, koldioxid och metan som gör det.

Det är ganska säkert att man kan mäta en höjning av medeltemperaturen på jorden, se fråga 15293 . Om höjningen är verklig kan det bero på vår påverkan, men det kan också ha naturliga orsaker. Typiskt svar från en vetenskapsman .

Några ovedersägliga fakta emellertid:

  1. Vår förbränning av fossila bränslen ökar koldioxidhalten i atmosfären med över 1 ppm per år (nuvarande värde är c:a 410 ppm (2018, se Carbon_dioxide_in_Earth%27s_atmosphere#Current_concentration ))
  2. Koldioxid är en effektiv växthusgas
  3. Klimatsystemet är mycker komplext med flera osäkra parametrar som har positiv eller negativ återkoppling

Vår påverkan bör alltså öka temperaturen; det osäkra är med hur mycket. Svårigheten är att atmosfären är ett mycket komplicerat system som påverkas av många faktorer. En liten ökning skulle kunna orsaka en stor ökning om den sätter igång en uppvärmande process.

Vattenånga är, som sagt, en viktig växthusgas. Mer vattenånga i atmosfären skulle betyda högre medeltemperatur på jorden. Men mer vattenånga kan betyda mer molnighet och därmed lägre instrålning. Detta skulle betyda lägre temperatur. Man kan alltså tänka sig att vattenångan har en stabiliserande verkan på temperaturen. Detta enkla exempel illustrerar hur komplext problemet med det globala uppvärmningen i själva verket är.

För vidare studier: artikel om växthuseffekten i Nationalencyklopedin (växthuseffekten ), Växthuseffekten från Svenska naturskyddsföreningen, Greenhouse_effect , Global_warming och nedanstående länkar.

Se även den trevliga videon från Lunds tekniska högskola:



/Peter E

Nyckelord: växthuseffekten [36]; jordens atmosfär [12];

1 http://www.zenker.se/Sv/istider_och_vaexthusgaser.shtml
2 http://hyperphysics.phy-astr.gsu.edu/hbase/thermo/grnhse.html

Avancerad sökning på 'växthuseffekt' i denna databas

*

Energi [13462]

Fråga:
Jag undrar om det finns en vettig förklaring till att Barsebäck ligger där det ligger?
/Caroline L, Tångvalla, Falsterbo

Svar:
Caroline! Barsebäck ligger där det ligger därför att det gick vägar dit och man tyckte det var lämpligt med några hus .

Men det var nog Barsebäcksverket du menade. Det är svårare. I dag anser nog många att placeringen inte är bra, men det ligger där det ligger. Man har försökt kompensera den dåliga läget med att öka säkerheten ytterligare genom att bygga en skorsten med filter som skall minska utsläpp vid en eventuell olycka.

Vad man gjort i detta fallet finns fler exempel på (bland annat i Frankrike på gränsen till Tyskland): man har bara tittat på vad som finns i omgivningen i det egna landet och glömt bort närliggande städer i utlandet. Man kan också uttrycka det så här: Skåne har glädje av Barsebäcksverket (energi, pengar) medan danskarna bara ser det som en fara och som en kunkurrent till den danska kolproducerade energin.

Så långt det objektiva svaret. Jag tänker nu kliva upp på min apelsinlåda och framföra mina högst personliga åsikter - du får tycka vad du vill om dem:

  • All energiproduktion påverkar miljön på något sätt. Man måste helt enkelt välja de metoder som är mest acceptabla. Detta är till sist ett politiskt beslut (skall så vara), men beslutet måste baseras på fakta och erfarenhet.
  • Kärnenergi är med tanke på alternativen en mycket bra energikälla - vi måste emellertid placera kärnkraftverken på rätt ställe. Jag kan nog hålla med så långt att Barsebäcksverket kan avvecklas på grund av politiska skäl (detta sker då tyvärr till ett högt pris, pengar som kunde använts till något annat), men vi bör samtidigt säga att vi skall behålla de övriga aggregaten ett bra tag till.
  • Alternativen till kärnkraft är framför allt vindkraft (rent, men fult och kan inte ge en stor del av vårt energibehov), vattenkraft (rent, men förstör våra älvar), direkt solenergi (ineffektivt i Sverige utom små anläggningar för att värma vatten, direkt elproduktion möjlig men dyr), kol/olja (moderna kraftverk ger inte mycket utsläpp av skadliga ämnen, men de bidrar till växthuseffekten genom CO2-utsläpp), fossilgas - felaktigt kallad naturgas (ger också CO2-utsläpp), biobränsle (bra på många sätt, men för mig åtminstone mest intressant som fordonsbränsle (rapsolja, sprit).
  • Vi kan också låta bli att producera elektricitet i Sverige och köpa från utlandet. Men då skyfflar vi bara över problemen till andra länder.
Det finns nya lovande metoder att få ut kärnenergi, se fråga 1351 . Med denna teknik undviker man de två problemen som finns med traditionell kärnkraft - säkerheten och avfallet. Se även länk 1 för miljöpåverkan och länk 2 för allmän information från Forskning.se.
/Peter E

Nyckelord: växthuseffekten [36]; kärnenergi [19]; energikällor [26]; *miljöpåverkan [14];

1 http://www.jordensvanner.se/2013/veckans-kronika-carl-erik-magnusson
2 http://www.forskning.se/nyheterfakta/teman/karnkraft/tiofragorochsvar/vadarkarnkraft.html

*

Universum-Solen-Planeterna [14214]

Fråga:
Milankovitch cykler
/Veckans fråga

Ursprunglig fråga:
Jag vill veta mer om Milankovich cykler; Vilka orsaker finns till att jordens bana runt solen ändras regelbundet likson lutningen av jorden egen axel? Förändringarna orsakar enligt Naturhistoriska Riksmuseets utställning 'Uppdrag Klimat' regelbundna naturliga klimatförändringar på jordklotet med istirder och värmeperioder. Jag vill veta mer!
/Katarina B, S:ta Ragnhildgymnasiet, Södertälje

Svar:
Milankovitch cykler är variationer av

1 Jordbanans excentricitet (hur avlång banan är)

2 Jordaxelns lutning mot normalen till jordbanans plan

3 Jordaxelns precession.

Förklaring för temperaturpåverkan för ovanstående

1 Om jordbanan är mycket avlång (hög excentricitet) blir klimatet kallare eftersom jorden tillbringar längre tid på ett avstånd från solen som är större än avståndet vid cirkulär bana. Detta beror på Keplers andra lag, se 12644 , som alltså säger att banhastighheten är lägre på större avstånd från solen. Jorden tillbringar alltså längre tid utanför medelavståndet, varför det blir kallare än när avståndet hela tiden är lika med medelavståndet. Observera att det är bara excentriciteten som ändras. Medelavståndet till solen (ellipsens halva storaxel) är konstant. Variationen i excentriciteten är komplex, med huvudkomponenten har en period på c:a 100000 år.

2 Till skillnad från 1 påverkar jordaxelns lutning inte den totala instrålningen utan bara fördelningen. Minskad lutning ger en större skillnad beroende på latitud: polerna blir kallare och områden nära ekvatorn blir varmare. Skillnaden syns bäst om vi betraktar extremerna. Om jordaxeln är vinkelrätt mot jordbanan kommer områden nära polerna att få mycket liten instrålning. Stora delar av polerna kommer att täckas av is. Om lutningen däremot är 0 (rotationsaxeln i jordbanans plan) kommer årstidsvariationerna att vara extrema, men eftersom jordens medeltemperatur är c:a 15oC, så kommer all is att smälta. Perioden för ändringen i jordaxelns lutning är c:a 41000 år och amplituden en grad. Se länk 1 och 2.

3 Jordaxelns precession orsakar en förskjutning i årstiderna (förutom att polstjärnan inte är polstjärna), se Precession_of_the_equinoxes#Effects . Perioden är 25772 år. För närvarande är jorden närmast solen i januari och längst ifrån i juli. Detta ger mindre extrema årstider på norra halvklotet. Efter 13000 år är jorden närmast solen i juli, så årstidsvariationerna på norra halvklotet förstärks.

Variationerna ovan beror på påverkan på jorden inte bara av solen utan även av månen och andra planeter. Den kombinerade effekten av dessa variationer ger upphov till ändringar i klimatet, t.ex. periodiskt återkommande istider.

Se även Milankovitch_cycles .

Påverkan på klimatet kompliceras av negativa och positiva återkopplingseffekter. N och S halvkloten är för det första olika. Nordpolen är hav och en övervägande del av norra halvklotet är kontinenter. Sydpolen är en kontinent och det mesta av södra halvklotet är hav.

Lösligheten av CO2 i havsvatten minskar med ökande temperatur, så CO2 flyttas upp i atmosfären vid förhöjd temperatur. Mer CO2 i atmosfären ökar växthuseffekten vilket leder till högre temperatur. Vi får alltså en förstärkning av effekten på temperaturen - en positiv återkoppling.

Is och snö har hög reflektionsförmåga (högt albedo). Det betyder att ett snötäckt område har mindre nettoinstrålning (instrålning-reflekterat ljus). Även detta är alltså en positiv återkoppling som förstärker temperatureffekterna.
/Peter E

Nyckelord: Milankovitch cykler [3]; istider [8]; växthuseffekten [36]; klimat [11];

1 http://www.ncdc.noaa.gov/paleo/milankovitch.html
2 http://www.planetseed.com/relatedarticle/sun-and-earth-and-temperature-change

*

Energi [14319]

Fråga:
Vad händer med den värmeenergi som kommer från tex en hårtork
/Veckans fråga

Ursprunglig fråga:
Hej! Jag undrar vad som händer med den värmeenergi som kommer från tex en hårtork. Värmer den upp vår jord eller vad blir det med den eftersom värme är den lägsta energiformen? Och en lite fråga till: Hur räknar man ut antal hästkrafter som en bil har?
/Sarah A, Lillerudsgymnasiet, Vålberg

Svar:
Sarah! Intressant fråga som är mer komplex än vad man kan tro! Det beror på var elektriciteten som driver hårtorken kommer ifrån.

Om den kommer t.ex. från vattenkraft, vindkraft eller solkraft så är det bara en omfördeling av värmeenergin. Den värme som skulle ha utvecklats vid kraftverket (och som kraftverket "stal") utvecklas i ditt sovrum i stället. Under största delen av året behöver vi i Sverige ändå värma upp våra bostäder, så värmeenergin är inte bortslösad.

Om el-energin kommer från kärnkraftverk är det lite annorlunda. Kärnkraftverken tar energi från urankärnorna och i slutändan blir all denna energi till värme. Utan kärnkraftverk hade urankärnorna bevarats och denna energi alltså stannat där. Energiutvecklingen från alla kärnkraftverk är emellertid mycket liten jämförd med den energi som kommer från solen. Låt oss se om detta är sant:

Enligt Power Reactor Information System är det totala elekriska effekten för alla världens kärnkraftevert 369 GW(e). Med en verkningsgrad på c:a 30% blir detta ungefär 1000 GW eller 1 TW (terawatt=1012 W) termisk effekt.

Instrålningen av energi per sekund och m2 från solen ges av solarkonstanten = 1368 W (se solarkonstanten ).

Jordens yta (genomskärningsytan) är

pR2 = p*(6.38*106)2 = 128*1012 m2

Effekten från solstålningen blir då

128*1012 * 1368 = 175000*1012 W = 175000 TW.

Detta är ganska mycket större är de 1 TW från kärnkraften, så uppvämningen av jorden pga kärnkraft är säkert försumbar - det finns andra viktigare skäl till att jordens temperatur antagligen ökar lite.

Se vidare energikällor , kärnenergi , vattenkraft , växthuseffekten . Se även fråga 14321.

Man bestämmer normalt effekten hos en bilmotor med en motor-dynamometer som är en sorts kalibrerad broms, se länk 1 för detaljer.
/Peter E

Se även fråga 14321

Nyckelord: energikällor [26]; kärnenergi [19];

1 http://www.pumaracing.co.uk/power3.htm

*

Universum-Solen-Planeterna [14416]

Fråga:
Lista på frågor om astronomi/astrofysik och rymdfart.
/Veckans fråga

Ursprunglig fråga:
Lista på frågor om astronomi/astrofysik och rymdfart.
/Peter E, Lunds universitet, Lund

Svar:
I vissa sammanhang kan det vara av värde att ha en sammanställning på frågor/svar inom ett visst ämnesområde. Här är länkar till nyckelord som har med astronomi/astrofysik och rymdfart att göra:

Astronomi/astrofysik
aberration
astrobiologi
astrologi
Big Bang
blå himmel
dagens längd
dagjämning
exoplaneter
galax
gamma ray burst
geodesi/gravimetri
gravitationsvågor
gravitationslins
gregorianska kalendern
grundämnen, bildandet av
HR-diagram
hägring
istider
jordens inre
jordens magnetfält
Keplers lagar
komet
kosmisk bakgrundsstrålning
kosmologi
kvasar
Lagrange-punkt
latitud/longitud
liv i universum
livets uppkomst
Mars
mass-luminositetsrelation
meteorit
Milankovitch cykler
månens bana
månens synbara storlek
mörk energi
mörk materia
neutronstjärna
Newtons gravitationslag
norrsken
nova
Olbers paradox
parallaxmetoden
Plancks strålningslag
planet
planeters atmosfär
planeters form
radioaktiv datering
radioteleskop
regnbåge
rödförskjutning
Saturnus´ ringar
SETI
solen, avstånd till
solaktivitet
solarkonstanten
solens energiproduktion
solens utveckling
solförmörkelse
solsystemet
solsystemet, avstånd
solsystemets bildande
stjärnhimlen
stjärnors utveckling
supernova
svart hål
teleskop
tidsekvation
tidvatten
universums expansion
Venus
vulkanism
växthuseffekten
årstider

Länkar: Cambridge Cosmology , Encyclopedia of Astronomy and Astrophysics , The Solar System in Pictures , Planetary Fact Sheets , Planetary Photojournal , Sky View Café , Nyhetsblogg för astronomi

Rymdfart
flykthastighet
geostationär satellit
gravity assist
Internationella rymdstationen
månfärder
raketekvationen
raketmotor
rymdfärder
tvillingparadoxen
tyngdlöshet
/Peter E

Nyckelord: *astronomi/astrofysik och rymdfart [2];

*

Energi [14591]

Fråga:
Vilka för- och nackdelar finns det med kärnkraft? Vilken framtid har kärnkraft, både i Sverige och omvärlden?
/karin h, ängskolan, solna

Svar:
Karin! Vi har svarat på massor av frågor om kärnenergi och energikällor .

Vad gäller framtiden så avser många länder (t.ex. Finland, Ryssland, Japan) bygga ut kärnkraften och flera (t.ex. EU, Storbritannien, USA) har börjat diskutera ny kärnkraft. Sverige, däremot, anser man kan vara utan både kärnkraft, ny vattenkraft och kol/olja/gas, hur det nu skall gå till . Förresten, vi vet hur det skall gå till: importera el från kärnkraft och kol från Finland, Polen och Danmark!

Nyligen (hösten 2009) har regeringssidan emellertid öppnat för att på sikt ersätta nuvarande 10 kärnkraftaggregat med nya som då skulle kunna ha högre effekt. Vi skulle då kunna fortsätta att ha i princip nollutsäpp av CO2 från produktionen av el. Transporter är emellertid det stora problemet med stora CO2-utsläpp.

Se vidare vattenkraft och växthuseffekten .
/Peter E

Nyckelord: energikällor [26];

*

Blandat [14739]

Fråga:
Jag håller på att skriva en uppsats om kyotoavtalet och klimatfrågan. En av mina källor är en kritisk text som menar att utsläppen inte minskar eftersom företag och regeringar köper trädplantage istället för att kompensera sina utslätt. Nu till min fråga, i denna text står det att:

"Koldioxid i träd slipper ur fångenskapen och återgår till atmosfären så fort träden dör sin naturliga död, eller om de huggs ner eller drabbas av skadeinsekter eller om skogsbrand bryter ut. Följaktligen kan kolsänkor i ett längre perspektiv omöjligen kompensera för att de uråldiga magasinen av koldioxid frigörs till atmosfären, där de stannar i växthuskupan i hundratals år."

Men trädet kan väl ändå inte suga upp all koldioxid och lagra det i sig? Träden omvandlar väl koldioxid till syre och kol? Och sedan om trädet brinner upp så blir det en liten kolhög, men det kan väl inte bli samma mängd koldioxid som trädet "sugit upp"? Eller har jag fått allt om bakfoten nu...
/Josefine B, Väggaskolan, Karlshamn

Svar:
Josefine! Nej, men lite har du fått om bakfoten .

Fråga 13757 beskriver den organiska CO2-cykeln. CO2 frigörs alltså när trädet brinner eller ruttnar. Å andra sidan binds CO2 när trädet växer. Så det jämnar ut sig i längden - därför ökar träd och andra nutida växter inte CO2-halten. Mycket gamla rester av växter (olja, kol, fossilgas) ger däremot upphov till en ökning.

Se även växthuseffekten .

Bilden nedan visar i vilken form kolet finns på jorden (från länk 1).



/Peter E

Nyckelord: koldioxidcykeln [6]; kol [3];

*

Ljud-Ljus-Vågor [14936]

Fråga:
Hur fungerar selektiva absorbenter i solfångare?
/Veckans fråga

Ursprunglig fråga:
I termiska solfångare finns s.k. selektiva absorbenter. Dessa sätter de vanliga formlerna för svartkroppsstrålning och för absorbtion och reflektion ur spel. Hur har man lyckats med denna bedrift. Kan man skapa en sådan yta med de resurser som finns i en ordinär gymnasiefysiksal? Om möjligt önskas ett utförligt svar eller hänvisning till litteraturen.
/Anders K, Sollefteå

Svar:
Nej, selektiva absorbenter sätter inte några naturlagar ur spel! Selektiva absorbenter innebär att ytskiktet på solfångaren behandlats med ett ytskikt som skall ge hög absorption av solstrålning och låg emittans av värmestrålning.

Kirchhoffs strålningslag säger att absorpionsförmågan är proportionell mot emissionsförmågan vid en viss våglängd. Solens yttemperatur är c:a 6000 grader, och den mesta energin i solstrålningen ligger i synligt ljus 400-700 nm. Det är alltså i detta område man vill ha maximal absorptionsförmåga hos en solfångare. Normalt innehåller en solfångare vatten som värmebärare, så temperaturen är maximalt 100 grader. Vid denna temperatur ligger maximum hos temperaturstrålningen vid mycket längre våglängder - i infrarött (se fråga 12793).

Med Blackbody Radiation Applet kan man uppskatta maximum i energifördelningen för olika temperaturer. För 6000 K ligger maximum vid 500 nm och vid 350 K (c:a 80oC) vid 8000 nm.

Även naturen utnyttjar denna selektiva absorption i växthuseffekten . Solljuset går obehindrat igenom atmosfären och värmer upp jordytan. Värmestrålningen från jordytan hindras att försvinna ut i rymden av växthusgaser - framför allt vattenånga och koldioxid. Utan denna värmande effekt skulle jorden vara c:a 35 grader kallare i medeltemperatur än vad den är.

Se vidare länk 1, solenergi , temperaturstrålning och Plancks strålningslag .
/Peter E

Se även fråga 12793

Nyckelord: solenergi [14]; Kirchhoffs strålningslag [4]; temperaturstrålning [29]; strålning, in-/ut- [6]; #ljus [63];

1 http://www.iva.se/upload/Verksamhet/Projekt/Energiframsyn/El%20och%20V%C3%A4rme%20komplett3.pdf

*

Energi [15136]

Fråga:
Man brukar prata om att avgaser från fossila bränslen genererar partiklar/ämnen som kan skada människans lungor. Bildas dessa partiklar/ämnen då man förbränner biobränslen tex vätgas/biogas/etanol? Skulle miljonstäder bli av med den smog som bildas av förbränning av fossila bränslen om de helt övergick till biobränslen?
/Roger S, Stockholm

Svar:
Aerosol är små partiklar som är finfördelade i en gas. Partiklarna kan endera vara fasta eller flytande, och aerosolen innefattar både gasen och partiklarna. Typiska exempel på aerosoler är rök, dimma och luftföroreningar. (Aerosol )

Aerosoler bildas vid all förbränning - om det är fossila bränslen eller biobränslen har ingen betydelse. Vedeldning ger t.ex. mycket partiklar. Vad gäller utsläpp av partiklar från fordon finns det även andra viktiga källor som t.ex. från dubbdäck och bromsar.

Man kan däremot minska utsläpp av partiklar med hjälp av filter. Det skulle kanske hjälpa lite mot smoggen, men inte helt eftersom en del smog kan bildas av molekyler som finns i avgaserna.

Den stora skillnaden mellan fossila bränslen och biobränslen är att de förra orsakar en ökning i koldioxiden i atmosfären, medan de senare inte ger någon nettoökning. Det är den viktigaste fördelen med biobränslen att de inte ger upphov till ökad växthuseffekt.

Bilden (från Wikipedia-artikeln Radiative_forcing ) visar olika bidrag till växthuseffekten*. Värdena på y-axeln, Radiative forcing) är ett mått på uppvärmningseffekten. För att få ett bättre mått på detta kan det vara bra att komma ihåg att 1 W/m2 motsvarar en uppvärmning på 0.8oC.

Även om felstaplarna är stora, så bidrager aerosoler antagligen till en avkylning. Detta är naturligtvis ingen lösning på uppvärmningsproblemet eftersom aerosoler är mycket skadliga att andas in. ____________________________________________________________________
* Radiative forcing is a measure of the influence a factor has in altering the balance of incoming and outgoing energy in the Earth-atmosphere system and is an index of the importance of the factor as a potential climate change mechanism. In this report radiative forcing values are for changes relative to preindustrial conditions defined at 1750 and are expressed in watts per square meter (W/m2). (IPCC)



/Peter E

Nyckelord: växthuseffekten [36]; fossila bränslen [13]; aerosol [4];

*

Energi [15293]

Fråga:
Hej! FN's rapport vissade ju att miljön var "dålig"! Vad i fysiken kan man bidra med för att försöka rätta till det här?
/Robin P, Centralskolan, Åtvidaberg

Svar:
Hej Robin! FN-rapporten behandlar framför allt våra utsläpp av koldioxid och den s.k. växthuseffekten . För att inte släppa ut alltför mycket koldioxid måste vi antingen spara energi eller använda källor som inte ger koldioxidutsläpp. Energiväxlande system (kärnkraftverk, vindkraftverk, förbränningsmotorer,...) baseras naturligtvis på fysikaliska principer, men de är så väletablerade tekniker att man knappast kan kalla det fysik. Kan vi hitta en ny fysikalisk effekt som kan utgöra en ny energikälla? Antagligen inte. Så problemet är egentligen inte fysikaliskt utan politiskt/ekonomiskt.

Däremot kan säkert fysiker vara användbara både för att effektivisera befintliga energikällor och energibärare och för att mäta och modellera komplexa system som t.ex. klimatvariationerna.

Bilden (från Wikipedia-artikeln Global_warming ) visar relativa uppmätta temperaturer under c:a 150 år. Det är svårt att förneka en ökning på c:a 0.7oC. Att denna ökning beror på människans verksamheter är de flesta överens om även om det naturligtvis som alltid finns avvikande åsikter.

Under länk 1 finns mycket omfattande temperaturdata.



/Peter E

Nyckelord: växthuseffekten [36]; *miljöpåverkan [14];

1 http://climate4you.com/

*

Blandat [15729]

Fråga:
Hej blev länkad hit när jag skulle skriva fråga om fysik, går i högskola. Läste en insändare på internet ang växhuseffekt. Där skrevs det att co2's temperaturhöjande effekt i atmosfären är logaritmiskt dvs minskar exponentielt med dess ökning. Är det så? Alltså att ju mer co2 det släpps ut dessto mindre blir temp ökningen.
/Marcus L, Jönköping tekniska, jönköping

Svar:
För en rimligt liten ökning är effekten linjär (allt är linjärt om man betraktar tillräckligt små bitar!). För mycket stora ökningar får man en mättnadeffekt när all strålning i CO2-banden absorberas. Då blir temperaturberoendet mer komplicerat och man parametriserar det ofta i klimatmodeller som logaritmiskt. En fördubbling av CO2-halten ger alltså en viss temperaturhöjning och en ytterligare fördubbling ger samma höjning.

Se vidare växthuseffekten och nedanstående länkar. Länk 1 är en relativt lättläst framställning av en enkel klimatmodell. I länk 2 (som är ganska avancerad) bestäms ett antal parametrar, bland annat tidskonstanten för global uppvärmning och ändring i temperatur för en fördubbling av CO2-halten. För det senare får man värdet 1.1+-0.5 K. Detta är alltså experimentellt bestämt från temperaturhöjningen 0.57+-0.08 K under 1900-talet, se fråga 15293.

Värdet c:a 1 K kan tyckas lågt, men anledningen är att förbränning som orsakar CO2-utsläpp även släpper ut aerosoler. Dessa orsakar en temperatursänkning - se diagrammet Radiative forcing i fråga 15136.

Det finns flera återkopplingar i det mycket komplexa systemet som bestämmer jordens klimat. Detta är anledningen till en viss oenighet eftersom det finns ett antal olika klimatmodeller som ger ganska olika resultat.
/Peter E

Se även fråga 15293 och fråga 15136

Nyckelord: växthuseffekten [36];

1 http://fragelada.fysik.org/resurser/climate_change.pdf
2 http://fragelada.fysik.org/resurser/schwartz.pdf

*

Energi [15732]

Fråga:
hej, vi håller på att forska om kolenergi, och vi lyckas inte hitta någon bra information på nätet. så därför undrar vi om ni skulle kunna förklara hur kolenergi fungerar? tacksam för svar
/Mikaela f, höglandsskolan, bromma

Svar:
Mikaela! Det är klart att det finns massor av information på nätet! Börja med Wikipedia-artiklarna nedan om fossila bränslen och kol som bränsle. Där finns även bra länkar.

Kort uttryckt får man kemisk energi genom att förbränna kol till koldioxid:

C + O2 -> CO2 + värme

Se även energikällor och växthuseffekten . Lycka till med forskningen!
/Peter E

1 http://sv.wikipedia.org/wiki/Fossila_br%C3%A4nslen
2 http://sv.wikipedia.org/wiki/Kol_%28br%C3%A4nsle%29

*

Blandat [16630]

Fråga:
Skulle utsläpp av vattenånga från vätgasdrivna bilar ge upphov till växthuseffekt?
/Veckans fråga

Ursprunglig fråga:
Hej! Det regnar och är grått ute. Och jag undrar om det med vätgasdrift av "alla" bilar skulle bli ännu varmare och regnigare, eftersom vattenånga är en växthusgas precis som koldioxid är? Så man skulle egentligen bara fylla på växthusgaserna med en annan än CO2.
/Thomas Å, Arlandagymnasiet, Märsta

Svar:
Hej Thomas! Som vanligt en relevant och klurig fråga!

Nej, utsläpp av vatten skulle inte ha någon märkbar betydelse eftersom det globalt inte är någon brist på vatten på jorden - 2/3 av ytan är ju täckt av hav! Det är andra faktorer som bestämmer hur mycket vattenånga det finns i atmosfären.

I applikationen under undervisningstips nedan kan man se att en fördubbling i den nuvarande CO2-halten ger mindre än en grads temperaturhöjning1). Om vi emellertid skulle ta bort all vattenånga skulle temperaturen sjunka med c:a 30 grader, och om vi dubblade halten vattenånga skulle temperaturen vara 25 grader högre än nu. Detta är naturligtvis den enklast tänkbara klimatmodellen, men visar att vattenångan är fundamental för förståelsen av den globala uppvärmningen.

Är det alltså vattenångan vi skall oroa oss för? I det temperaturintervall vi har på jorden i dag förekommer vatten som ånga (gasform), moln (vattendroppar, iskristaller), öppet vatten, snö och is. Övriga växhusgaser ha ingenstans att ta vägen. De är i gasform vid alla rimliga temperaturer, och utbytet med haven är ganska långsamt. Vattenånga däremot kan kondenseras och falla ner som regn och på så sätt försvinna som växthusgas. Vattenånga kan bilda moln som ökar reflektionsförmågan, vilket minskar instrålningen och temperaturen minskar. Om det bildas snö och is på marken eller på havsytan, kommer också reflektionsförmågan att öka med minskad temperatur som följd. Det är som synes mycket komplicerat att bestämma om vattnet dämpar eller accelererar den globala uppvärmningen.

Engelska Wikipedia säger om detta (Water_vapor#Water_vapor_in_Earth.27s_atmosphere ):

Gaseous water represents a small but environmentally significant constituent of the atmosphere. Approximately 99.99% of it is contained in the troposphere. The condensation of water vapor to the liquid or ice phase is responsible for clouds, rain, snow, and other precipitation, all of which count among the most significant elements of what we experience as weather.

Less obviously, the latent heat of vaporization, which is released to the atmosphere whenever condensation occurs, is one of the most important terms in the atmospheric energy budget on both local and global scales. For example, latent heat release in atmospheric convection is directly responsible for powering destructive storms such as tropical cyclones and severe thunderstorms. Water vapor is also a potent greenhouse gas.

Because the water vapor content of the atmosphere is expected to greatly increase in response to warmer temperatures, there is the potential for a water vapor feedback that could amplify the expected climate warming effect due to increased carbon dioxide alone. However, it is less clear how cloudiness would respond to a warming climate; depending on the nature of the response, clouds could either further amplify or partly mitigate the water vapor feedback.

I nedanstående diagram från Wikimedia Commons (Greenhouse_gas#Role_of_water_vapor ) syns en klar trend att halten vattenånga i stratosfären ökas. Nedre delen av figuren visar ökningen under en 22-årsperiod som funktion av höjden.

Ökningen beror antagligen på att atmosfärens temperatur har ökat, så att den kan hålla mer vattenånga. Om nettoeffekten av mer vattenånga ger en ytterligare ökad temperatur (genom att vattenångan fungerar som växthusgas) eller en minskad temperatur (genom ökad molnbildning) är som sagt osäkert.

Undervisningstips

Länk 1 innehåller applikationen Surface Temperature of Terrestrial Planets (Jordliknande planeters yttemperatur). Det är en mycket bra "tutorial" om hur temperaturen på en planet beror av olika parametrar. Applikationen visar mycket bra de viktigaste faktorerna som bestämmer medeltemperaturen på en planet: avstånd till solen, reflektionsförmåga (albedo) och förekomst av växthusgaser. Relevant för studier i astrobiologi men även mycket relevant för diskussion av växthuseffekten .

______________________________________________________
1) Detta kan tyckas konstigt när man talar om att en relativt marginell ökning av CO2-halten under några tiotal år skulle ge flera graders temperaturökning. Förklaringen är att atmosfären är ett mycket komplext system med många komponenter (se fråga 15136 ) som är kopplade och både kan förstärka och motverka temperaturhöjningen.



/Peter E

Nyckelord: växthuseffekten [36];

1 http://media.pearsoncmg.com/bc/bc_bennett_essential_2/tutorials/PlanetST/GoPlanetST.html

*

Energi [16846]

Fråga:
Om klimatmodeller
/Veckans fråga

Ursprunglig fråga:
0. Hur visar man att den s.k solarkonstanten är 1340W/m upphöjt till två , som t.ex NE påstår? Vad menas egentligen med solarkonstanten, vad används den till?

1. a).Om jorden antas vara en svart kropp utan atmosfär som sväljer all inkommande strålning, och strålar ut enligt stefan.boltzmanns lag, hur räknar man fram jordens medeltempratur vid jämvikt? Hur gör man detta enklast? Och hur visar man det på enklast pedagogiska vis med en "figur"? b).Om 30% av strålningen sedan reflekteras utan att påverka dvs att jordens albedo är 30%, hur visar man det då?

2. En del av den strålning som jorden avger tas upp av atmosfären. Varför kan den tas upp men inte den infallande? Antag att atmosfären tar upp all den strålning som jorden avger. Vid jämvikt kommer atmosfären att avge lika mycket strålning, denna strålning avges både uppåt och nedåt. Antag vidare att jorden tar upp det nedåtriktade bidraget från atmosfären och använd för att beräkna ny medeltemp för jorden?

3. För att som i uppgifter ovan illustrera en mer verklighetstrogen bild av jorden och dess atmosfärs uppbyggnad, vad bör man ta med i en sådan modell?
/Amanda S, Galären, Karlskrona

Svar:
0 Amanda! Det finns flera frågor som behandlar solarkonstanten , se t.ex. fråga 13917 . Värdet är 1370 W/m2.

1 För konstant temperatur:

Instrålad effekt = utstrålad effekt

där vänstra ledet är

(1-a) * (solarkonstanten) * pR2

R är jordradien och uttrycket ovan är den yta som träffas av strålningen, a är jordens albedo (reflektionsförmåga, dvs hur stor del av den inkommande strålningen som direkt reflekteras tillbaka ut i rymden). Jorden medelalbedo är c:a 0.36, så 0.64 av strålningen absorberas och ger bidrag till uppvärmningen.

Den utstrålade effekten är från Stefan-Boltzmanns lag (Stefan-Boltzmann_law )

4pR2*sT4

(observera att här har vi hela klotets yta!)

Vi får alltså

(1-a) * (solarkonstanten) * pR2 = 4pR2*sT4

dvs

(1-a)*(solarkonstanten) = 4sT4

T4 = (1-a)*(solarkonstanten)/(4*s) = 0.64*1370/(4*5.67 10-8) = 38.7 108

dvs

T = 249 K = (249-273) = -24oC

Jämfört med jordens uppmätta medeltemperatur, c:a 15oC, är detta mycket lågt. Observera dock att vi ännu inte tagit hänsyn till växthuseffekten .

2 Växthuseffekten behandlas detaljerat i fråga 12668 . Anledningen till att infallande och utgående strålning påverkas olika av atmosfären är helt enkelt att de innehåller helt olika våglängder. Det infallande spektret (vita pilar i nedanstående figur från länk 1) är temperaturstrålning från en kropp med temperaturen 5700 K (solen), vilket ger maximum för synligt ljus. Den utgående strålningen (röda pilar) kommer från en kropp med mycket lägre temperatur, varför maximum ligger i infrarött/mikrovågor. Om den utgående strålningen hindras får man en obalans i effekten och jordens temperatur ökar. Detta betyder att utstrålningen ökar tills balans nås och vi fått en högre jämviktstemperatur. Den största delen av temperaturhöjningen pga växthuseffekten kommer från vattenånga i atmosfären. Koldioxiden bidrar direkt till en mycket liten höjning, se länk 1.

3 Verkligheten är i själva verket mycket mer komplicerad. Dels finns det fler faktorer att ta hänsyn till (t.ex. aerosoler) och dels har man återkoppling mellan de olika parametrarna. Om man t.ex. ökar värdet på en parameter kan det tänkas att en annan parameter antingen minskar eller ökar. Detta är skälet till att olika klimatmodeller ger olika resultat.

Länk 1 är en mycket bra övning vad gäller temperaturbalansen. Solar_radiation_management behandlar möjliga metoder att påverka jordens temperatur genom att ändra albedot (geoengineering, se länk 2). Climate_model är en bra introduktion till klimatmodeller.



/Peter E

Nyckelord: solarkonstanten [6]; växthuseffekten [36]; jordens atmosfär [12]; klimat [11];

1 http://media.pearsoncmg.com/bc/bc_bennett_essential_2/tutorials/PlanetST/GoPlanetST.html
2 http://fragelada.fysik.org/resurser/geoengineering.pdf

*

Energi [17071]

Fråga:
Hej,

Jag såg en film på youtube där en man ringer in till "ring P1" (2008-12-01).

Man säger att solceller bidrar till växthuseffekten eftersom att de 80% som inte blir energi blir värme. Det låter ju som strunt. Dessutom säger han att något om att den belysta ytan ökar om du har en solcell med en reflektor, jämfört med platt mark. Det låter väl rimligt, men då borde det väl bli mindre energi per ytenhet?

Han blir då bemött med att "den här energin fanns ju där från början". Annars skulle det väl vara nån sorts evighetsmaskin. Men jag kan ändå inte komma på hur man ska tänka för att förklara var resonemanget brister. Kan ni hjälpa mig?
/Peter L, Naturhumanistiska gymnasiet, Malmö

Svar:
Hej Peter!

Ring P1 är en samlingsplats för "nutties", och jag slår genast över till P4 när Ring P1 börjar. Den intellektuella nivån i P4:s frågesportprogram är einstein-nivå jämfört med Ring P1!

Du har helt rätt att uppringaren pratar nonsens. Vad gäller den låga verkningsgraden så är det inget problem eftersom det på rätt plats finns massor av energi i solljuset (se kommentaren om verkningsgrad i fråga 17042 ). Resonemanget att de 80% som inte blir elekticitet ger en extra grobal uppvärmning är inte sant: all solinstrålning på en yta ger uppvämning, se fråga 16846 . Möjligen kan reflektionsförmågan (albedo) hos en solcell vara lite mindre än medelvärdet för jorden. Men ytan som solcellerna upptar är mycket liten. I princip skulle man kunna kompensera ändringen i albedo med vita ytor.

Ett par citat från inslaget: "du ökar aperturarean den belysta ytan med en veckad sofångare" och "om du tar en kvadratmeter och lägger dit en solfångare på två kvadratmeter så blir det dubbelt så varmt".

Det är klart att veckningen inte har någon betydelse. Den maximala effekten man kan få ut är den effekt som motsvarar tvärsnittsytan vinkelrätt mot solen. Detta är ju anledningen att det är kallare i Sverige än vid ekvatorn: solljusets effekt sprids över en större yta. Dessutom är uttrycket "dubbelt så varmt" fullständigt nonsens!

När det gäller storskalig elektricitetsproduktion är solkraftverk med speglar som koncentrerar solljuset, värmer vatten som driver en generator en mycket lovande teknik som redan finns bland annat i Spanien i kommersiell drift, se Solar_power#Concentrating_solar_power och nedanstående bild från Wikimedia Commons.



/Peter E

Nyckelord: solenergi [14]; växthuseffekten [36];

*

Ljud-Ljus-Vågor [17228]

Fråga:
Hej, Jag vet att himlen är blå på grund av Rayleighspridningen. Som jag förstår det har luftens molekyler resonansfrekvenser i det ultravioletta området (men inte i det infraröda), vilket leder till att (synligt) ljus med kortare våglängd exiterar dem mer än ljus med längre våglängd.

Min fråga är. I en teoretisk atmosfär där koldioxid är den dominierande gasen. Skulle solnedgångarna vara blå då? Och himlen röd?

Är spridningen i koldioxid samma mekanism som Rayleigh spridning i vanlig luft.
/David Ö, Uppsala

Svar:
David! Vi har nog sagt vad som kan sägas om detta ämne, se fråga 13750 , 14360 och 14679 . Vilken gas det är har alltså lite betydelse. Stoft och aerosoler påverkar emellertid himlens färg.

Följdfråga:
Om gasens typ inte spelar roll för rayleigh spridningen, varför är i så fall vissa gaser växthusgaser men andra inte.

Det är ett helt annat problem, se fråga 12668 . Rayleigh spridningen vi talar om är blått ljus, alltså strålning i det synliga området. För växthuseffekten är strålkällan (jorden) c:a 300 K, och strålar i infrarött-mikrovåg området. För dessa senare våglängder spelar vibrations- och rotationstillstånd i molekylerna stor roll. Tvåatomiga, symmetriska molekyler (N2 och O2) saknar dessa tillstånd i rätt energiområde, så de är inte växthusgaser.

Det är alltså skillnaden i temperatur mellan solen (c:a 5700 K) och jorden (c:a 300 K) som gör att den inkommande och den utgående strålningen är av så olika energi, och följaktligen påverkas mycket olika av atmosfären, se figuren i fråga 12668 .
/Peter E

Nyckelord: växthuseffekten [36];

*

Universum-Solen-Planeterna [17484]

Fråga:
När svalnar jordens inre av?
/Veckans fråga

Ursprunglig fråga:
Om jorden svalnade av helt så skulle kontinentaldriften stanna och med tiden berg och land nötas ner/eroderas av regn och vind och vatten täcka hela planeten har det sagts mig; den Blå planeten är ett faktum. Det verkar rimligt. Men vilka tidrymder skulle det handla om, dvs hur länge kommer jorden att ha ett varmt, drivande inre och hur lång tid kan erosionen antas ta därefter?

Månen lär ha bildats ungefär samtidigt som jorden. Har den också en varm kärna? Eller har kärnreaktionerna tagit slut där; slut på bränsle helt enkelt?
/Thomas Å, Knivsta

Svar:
Thomas! Det är radioaktivt sönderfall som håller jordens inre flytande, se fråga 13938 . Eftersom halveringstiderna är över 1 miljard år, är avsvalningen inget vi behöver oroa oss för på en tidsskala 100 miljoner år. Sedan tror jag inte jorden skulle bli en havsplanet. Om kontinentadriften upphörde skulle koldioxidhalten i atmosfären öka - fråga 17321 beskriver processen som håller nere koldioxidhalten - och växthuseffekten skulle orsaka en temperaturökning liknande vad som skett med Venus. Med en temperatur på flera 100 grader Celsius kan det inte förekomma flytande vatten.

Vad gäller månen så är den redan avsvalnad - den innehåller ingen flytande kärna. Anledningen är inte att det finns mindre halt av radioaktiva element utan att månen är mindre. Uppvämningen från radioaktivitet är proportionell mot massan som är proportionell mot volymen = 4pr3/3. Förlusten av energi genom elektromagnetisk strålning (värmestrålning) är proportionell mot ytan = 4pr2. Förhållandet mellan uppvärmning och avsvalning är alltså proportionell mot radien r. För liten radie dominerar utstrålningen och man får en låg jämviktstemperatur, se fråga 13938 hur man räknar ut denna.
/Peter E

Nyckelord: jordens inre [14]; *geologi [16];

*

Blandat [17975]

Fråga:
Hej! I tidningen nämns att man nu ser att havsytan stiger 3 mm per år, medan den under 1900-talet steg 1 mm per år. Den snabbare höjningen antas bero på koldioxidhaltens ökning/växthuseffekten/klimateffekten. Men vilken var anledningen till den tidigare höjningen? Utan klimateffekter borde ingen höjning alls ha noterats väl? (Visserligen ingen direkt fysikfråga, men fysiken/-rna brukar ha bra svar på det mesta!)
/Thomas Å, Knivsta

Svar:
Thomas! Vi har svar på allt!

Figuren i fråga 15293 visar uppmätt medeltemperatur för jorden. Det finns antydan till klimateffekter redan från början av 1900-talet. Figuren nedan från Wikimedia Commons (Current_sea_level_rise ) visar en ganska konstant stigning av havsytan från 1900 med i medeltal 20/100 = 0.2 cm/år = 2 mm/år. Från andra data (Wikipedia-artikel ovan) finns indikationer på en liten ökning till 3 mm/år.

Stigningen orsakas alltså av temperaturhöjningen (som i sin tur beror på växhuseffekten) dels för att istäcken på land smälter (Grönland, Sydpolen) och dels för att vattnet expanderar med stigande temperatur.



/Peter E

Nyckelord: växthuseffekten [36];

*

Blandat [18635]

Fråga:
Jordens strålningsbalans
/Veckans fråga

Ursprunglig fråga:
Hej,

När man räknar på jordens strålningsvikt och undantar växthuseffekten, är två vanliga antaganden som följer:

1. Jorden absorberar ca 70% av den infallande strålningen från solen.

2. Jorden strålar som en svartkropp.

I mina ögon motsäger dessa två antaganden varandra. Om jorden endast absorberar 70% av den infallande strålningen så är den ju bevisligen ingen svartkropp. Vore det då inte rimligt att räkna med att jorden till 70% strålar som en svartkropp; d.v.s. att vi modifierar Stefan-Boltzmanns lag med en faktor 0,7. Räknar man dock på detta senare sätt spelar det ingen roll hur många procent av strålningen som jorden absorberar, eftersom dessa faktorer 0,7 tar ut varandra. Följden av detta blir att beräknad temperatur blir ca 6 grader C; vilket är alldeles för varmt (utan hänsyn tagen till växthuseffekten ska ju jordens temperatur landa på ca -15 grader C). Var tänker jag fel?
/Patrik F, Södra Latins gymnasium, Stockholm

Svar:
Patrik! Bra fråga. Som du anar så tänker du fel.

Instrålningen från solen har sitt maximum i det synliga området (400-700 nm) eftersom solen har en temperatur på c:a 5800 K. Utstrålningen från jorden har sitt maximum i infrarött/mikrovågor eftersom temperaturen är c:a 300 K.

Maxvåglängden ges av Wiens förskjutningslag, se fråga 12397 . Låt oss beräkna våglängderna för maximum för dessa temperaturer:

Solen, T=5800 K
lmax = 2.898×10−3/T = 2.898×10−3/5800 = 500 nm

Jorden, T=300 K
lmax = 2.898×10−3/T = 2.898×10−3/300 = 9.7 mm

Emissiviteten (e i Stefan-Boltzmanns lag, fråga 12397 ) är en funktion av våglängden och relaterad (lika med) absorptionsförmågan, se fråga 9333 . Eftersom jorden och solen sänder ut strålning vid vitt skilda våglängder, så finns det ingen anledning att emissiviteten (och absorptionsförmågan) skulle vara densamma.

Att jorden och solen strålar i helt olika våglängdsområden förklarar alltså varför strålningen från solen (ljus) kan ta sig igenom atmosfären medan strålningen från jorden (infrarött) stoppas av atmosfären, se figuren i fråga 12668 .

I fråga 16846 beräknas jordens temperatur från strålningsbalansen. Fråga 17681 behandlar en liknande fråga om vita värmeelement. I fråga 14936 behandlas selektiva absorbanter.
/Peter E

Nyckelord: växthuseffekten [36]; temperaturstrålning [29];

*

Blandat [18867]

Fråga:
Har den globala uppvärmingen räddat oss från en ny istid?
/Veckans fråga

Ursprunglig fråga:
Hej! I ett diagram i en dagstidning visade man att medeltemperaturen på jorden sjunkit c:a 0,4 grader från år 0 till c:a år 1900. Under 1900-talet har temperaturen ökat kraftigt, och det antas vara p g a mänskligt inflytande.

Tyder det på att utan mänskligt inflytande så skulle vi ha gått mot en ny istid om ett par tusen år? Istiderna lär visst pågå 110 kiloår och mellanistiderna bara i 10 kiloår, så det vore ju då snart dags för en ny; den senaste slutade ju ett tag före vår stenålder. När under mellanistiderna är det som varmast? Finns regelbundenhet?
/Thomas Å, Knivsta

Svar:
Thomas! Jag vet inte vilken figur du syftar på, men jag antar det var en lik nedanstående från Global_warming#Observed_temperature_changes . Plotten är från data från historiska källor, trädringar, glaciärer, koldioxidhalt, mm.

Osäkerheten är emellertid stor, och det är inte säkert att kurvorna representerar globala temperaturer. Dessutom är temperaturändringarna mycket små. Förutom den mycket snabba ökningen från 1900 (ganska säkert beroende på vår användning av fossila bränslen) är det två effekter som är någotsånär etablerade:

1 Den medeltida varma perioden (Medieval_Warm_Period ).

2 Den lilla istiden (Little_Ice_Age ). Denna är egentligen inte en riktig istid utan en period med speciellt kalla vintrar eventuellt med samband med den låga solaktiviteten (Maunder_Minimum#Little_Ice_Age ).

Enligt figuren i fråga 830 varierar de varma perioderna mellan 10000 och 20000 år, men de olika 110000-årsperioderna har ganska olika struktur, så man kan knappast dra någon slutsats av dem. Men det är korrekt att mesta tiden är istider.

Om den globala uppvärmningen orsakad av växthuseffekten räddat oss från nästa istid kan vi inte säga något om. Frågan är vilket som är värst: att dränkas av höjda havsnivåer eller att täckas av ett flera kilometer tjockt istäcke! I vilket fall som helst så sker ändringarna långsamt (100-1000-tals år), så vi hinner flytta. Tidsskalan för uppvärmningen p.g.a. fossila bränslen är emellertid mycket kortare än tidsskalan för istiderna, så min gissning är att vi inte får någon mer istid på ett bra tag.

Den lilla istiden är för övrigt möjligen orsaken till att skåningar är svenskar och inte danskar -- Karl X Gustav med armé kunde tåga över bälten och besegra danskarna (freden i Roskilde 1658), se länk 1, 2 och Tåget_över_Bält .



/Peter E

Nyckelord: istider [8]; växthuseffekten [36]; klimat [11];

1 http://klimat.wordpress.com/2006/03/01/vintern-som-skapade-stormaktssverige/
2 http://www.global-warming-and-the-climate.com/mann's-hockey-stick-climate-graph.htm

*

Energi [18901]

Fråga:
1.Vilka ämnen bidrar till att växthuseffekten är ett miljöproblem?

2.Vilka konkreta åtgärder anser du att vi i vårt samhälle måste vidta för att minska den globala uppvärmningen? Välj ut 3 saker som du själv skulle kunna göra.
/Oskar I, Strömbacka, Piteå

Svar:
Oskar! Det låter som ett projekt som du skall arbeta på själv, så svaret blir kort.

1 Vatten, koldioxid och metan är de viktigaste, se Växthusgas .

2 Du som individ kan bäst bidra med att påverka politiker att med regleringar och satsningar på koldioxidfria alternativ för energiproduktion minska användningen av fossila bränslen.

Läs bakgrund i tidigare frågor (växthuseffekten ).

Lycka till!
/Peter E

*

Blandat [19099]

Fråga:
jag kan härleda jordens temperatur utan växthuseffekt, men hur använda avstånden i solsystemet i härledningen? tacksamt om jag får hjälp med frågan.
/maria k

Svar:
Temperaturen härleds från solarkonstanten i fråga 16846 .

Solarkonstanten beräknas från solens effekt och jordens avstånd från solen i fråga 13917 .
/Peter E

*

Blandat [19124]

Fråga:
Hur kan man rita en tankekarta över fysik och jordens klimat. t.ex vad påverka? Vad behöver man ta hänsyn till i klimatmodellen
/maria k, Göteborg

Svar:
Fysik och jordens klimat låter ganska omfattande. Det är klart man kan göra en tankekarta, men det det bör du göra med några kollegor - det är snarare vägen än målet som är det viktiga.

Det finns naturligtvis en hel del fysik i klimatfrågan. Här är ett urval nyckelord där vissa svar kan vara relevanta:

istider
växthuseffekten
temperaturstrålning
koldioxidcykeln
jordens atmosfär
solenergi
vindenergi
solarkonstanten
miljöpåverkan
fossila bränslen
aerosol
kärnenergi
energikällor

Det finns massor på webben om växthuseffekten och klimatmodeller, t.ex. länk 1, 2 och Climate_model .
/Peter E

1 http://occri.net/climate-science
2 http://hyperphysics.phy-astr.gsu.edu/hbase/thermo/grnhse.html#c1

*

Blandat [19176]

Fråga:
Vad har den vulkaniska aktiviteten har betytt för livets uppkomst?
/Veckans fråga

Ursprunglig fråga:
Jag undrar på vilket sätt "livet på jorden" har vulkanismen att tacka och vad egentligen den vulkaniska aktiviteten har betytt för jorden uppkomst?
/Emelie A, Lars Kaggskolan, Kalmar

Svar:
Mja, kanske inte direkt men kombinationen hav och kontinentaldriften har en stabiliserande verkan på koldioxidhalten och därmed på jordens temperatur, se fråga 17321 .

Venus, som saknar hav och kontinentaldrift har en atmosfär med koldioxid av 90 atmosfärers tryck och därmed, genom en extrem växthuseffekt, en yttemperatur på uppemot 500oC, en temperatur som omöjliggör liv som vi känner det.

Det är möjligt att domänen arkéer (se Arkéer ) är ursprunget till liv i s.k. "black smokers" (se Hydrothermal_vent ).

Vulkanism har även antagligen orsakat massutdöenden som stimulerat utvecklingen av nya arter (se Massutdöende ).
/Peter E

Nyckelord: *geologi [16]; Venus [11]; växthuseffekten [36]; vulkanism [5]; *biologi [20];

*

Universum-Solen-Planeterna [19301]

Fråga:
Vad skulle hända om jordens inre stelnade och blev hård?
/Veckans fråga

Ursprunglig fråga:
Vad skulle hända om jordens inre stelnade och blev hård?
/Mary D, Skälltorpskolan, Hisings/backa

Svar:
Mary! Se fråga 17056 och 17484 varför jordens inre är varmt och flytande. Temeraturen är 1000-7000 K beroende på djup, se nedanstående figur från Geothermal_gradient .

Det omedelbara som skulle ske om jordens inre stelnade är att vi inte skulle få några jordbävningar eller vulkanutbrott. Det är kanske bra, men värre är att jordens magnetfält (fråga 18768 ) skulle försvinna.

Förutom att kompasser skulle bli förvirrade så skulle avsaknad av magnetfält göra att vi inte har något skydd för partikelstrålning från solen, se Solvind . Detta osakar en ökad nivå av joniserande strålning och en gradvis uttunning av atmosfären.

Om jorden inre stelnar kommer även kontinentaldriften att upphöra. Denna har en stabiliserande verkan på klimatet genom att hålla koldioxidhalten i atmosfären på en låg nivå, se fråga 17321 . Utan kontinentaldrift skulle jorden kunna råka ut för en extrem växthuseffekt som planeten Venus med medeltemperatur på uppemot 500oC.

Nu är dessa effekter ingenting att oroa sig för eftersom jordens inre kommer att fortsätta att vara flytande under hundratals miljoner år framåt.



/Peter E

Nyckelord: *geologi [16]; jordens inre [14]; jordens magnetfält [22];

*

Universum-Solen-Planeterna [19751]

Fråga:
vad händer om solen kommer närmre jordklotet?
/sara B, osby

Svar:
Eftersom solen är mycket större än jorden är det bättre att säga att jorden kommer närmare solen.

Låt oss först fråga oss om det är sannolikt att jordens medelavstånd till solen ändras. Detta behandlas i fråga 17160 och slutsatsen är att planeterna banor är mycket stabila speciellt vad gäller medelavstånden till solen. Om större förskjutningar sker så tar det mycket lång tid (miljarder år).

Om jorden kommer närmare solen kommer temperaturen att stiga. Om temperaturen ökar tillräckligt kommer vattnet att förångas, vilket genom växthuseffekten ger en ytterligare temperaturökning. Effekten förstärks om karbonat-silikatcykeln, se fråga 17321 , stoppas. Då kommer CO2-halten att öka. Man får vad som kallas skenande växthuseffekt(se Runaway_greenhouse_effect ). Planeten Venus har råkat ut för detta med en yttemperatur på 500oC och 90 atmosfärers tryck av CO2. Om detta skulle ske med jorden skulle den bli obeboelig. Nu fordras det en ganska stor temperaturökning för att karbonat-silikatcykeln skall stoppas, så det är inget vi behöver oroa oss för.
/Peter E

Nyckelord: solsystemet [8]; växthuseffekten [36]; koldioxidcykeln [6];

*

Ljud-Ljus-Vågor [21464]

Fråga:
Hej,vad är det som gör att ett material kan skärma av ur strlålning? vi ska skriva en labbrapport på ett experiment där man har en fjärrkontroll och använder olika material för att skärma av it strålningen. vi ska sedan försöka lista ut vad det är som gör att materialet stoppar eller inte stoppar it strålningen, vilket jag inte kan lista ut. är det tex genomskinlighet, densitet eller hur bra materialet leder värme? tack för svar!
/Amanda V, Nannaskolan, Uppsala

Svar:
Vad gäller absorption av strålning så varierar fysiken mycket beroende på våglängden. Eftersom ni använt en fjärrkontroll så är det här fråga om infrarött som har våglängder på 700 nm till 1 mm.

För att ett material skall absorbera strålning måste den växelverka med atomer, molekyler eller fasta ämnen. För synligt ljus kan detta ske genom atomernas elektroner.

För infrarött räcker energin ofta inte till för växelverkan med elektroner. I stället exciteras molekylers vibrations- och rotationstillstånd som har lägre energi.

Vatten- och koldioxidmolekyler absorberar infrarött (och mikrovågor - tänk på växthuseffekten!). Hög densitet ökar naturligtvis absorptionsförmågan. Förekomsten av fria elektroner (metaller) ger också hög absorption eller reflektion.

Se vidare fråga 21018 .

Se även infrared .
/Peter E

Nyckelord: elektromagnetisk strålning [21];

1 https://sciencing.com/materials-absorb-infrared-rays-8044395.html

*

[19997]

Fråga:
1.Hur uppstår en vind? Hur uppstår sjöbris och landbris? 2.Förklara vad följande klimattyper är: Tropiskt klimat Subtropiska zonen Tempererat klimat Polarklimat 3.Välj ett väderfenomen tropiska och ett land som väderfenomenet finns i och svara på nedanstående frågor. 4.Varför uppstår väderfenomenet? Vilka egenskaper har väderfenomenet (nederbörd, vindstyrka)? Hur påverkas jordbruk och övrigt näringsliv av väderfenomenet? Hur påverkas hela landet om väderfenomenet slår till? Vad kan människor göra för att minska effekterna av väderfenomenet? Hur påverkar olika faktorer hur hårt ett land/område drabbas av ett väderfenomen? Ge exempel från olika delar av världen och resonera utifrån följande: Levnadsstandard, om landet är rikt eller fattigt Infrastruktur, hur vägar, hus och kommunikationer är byggda Näringar, vad man arbetar med 4.Beskriv växthuseffekten. Beskriv och förklara människans påverkan på klimatet. Vad kan göras för att minska utsläppen av växthusgaser?
/Mary D, Jensen, Hisings backa

Svaret kommer snart...

*

Universum-Solen-Planeterna [20222]

Fråga:
Hej! De stora planeterna påverkar jordens bana periodiskt så att istider är "normala" på jorden. Finns samma påverkan och fenomen belagda på mars resp venus?
/Thomas Å, Knivsta

Svar:
Här är lite om hur banorna ändras: 17160 . Framför allt banans excentricitet (avlånghet) påverkar temperaturen. För Mars och Venus finns det inte några data liknande de för jorden (830 ) för storleksordningen ett antal 100-tusen år eftersom man inte har geologiska data som för jorden -- man har hittills bara krafsat lite på ytan.

Från början (flera miljarder år sedan) bör Venus, jorden och Mars ha haft liknande atmosfärer. Venus drabbades av en skenande växthuseffekt och fick en mycket tät koldioxidatmisfär med uppemot 500 graders temperatur. Mars har antagligen haft hav med vatten, men dessa har försvunnit med atmosfären. Det är möjligt att man kan bestämma temperatur och atmosfärens sammansättning genom att som på jorden analysera bubblor i polarisen. För detta behöver vi nog åka dit.
/Peter E

Nyckelord: istider [8]; Mars [12]; Venus [11];

*

Universum-Solen-Planeterna [20231]

Fråga:
Varför är Venus', jordens och Mars' atmosfärer så olika?
/Veckans fråga

Ursprunglig fråga:
Hej! Solvinden tär på jordens atmosfär, såg jag i en skrift, så att atmosfären blir tunnare, om än långsamt. Samma sak har väl då hänt med Mars och borde ha hänt med Venus, speciellt med Venus, som ligger närmare solen och utsätts för starkare vind. Ändå lär lufttrycket vara större på Venus. Hur kommer det sig?
/Thomas Å, Knivsta

Svar:
Anledningen till att jordens atmosfär innehåller så lite koldioxid är att kontinentaldriften för ner, bland annat, i havet sedimenterat CaCO3 till jordens inre (se fråga 17321 ). Denna transport är i jämvikt med kodioxid som frigöres vid vulkanutbrott. Vår förbränning av fossila bränslen (olja, kol, fossilgas) stör jämvikten så att koldioxidhalten och därmed jordens temperatur ökar. Det är detta som kallas global uppvärmning.

Mars är ganska liten och saknar magnetfält som skyddar atmosfären från solvinden. Mars förlorade sin atmosfär ganska tidigt, se Mars#Atmosphere . För flera miljarder år sedan hade Mars antagligen hav av vatten, men de försvann med atmosfären.

Venus (se bilden nedan) atmosfär är emellertid helt annorlunda: mest koldioxid med ett tryck på nära 100 atmosfärer. Detta ger en extrem växthuseffekt med en yttemperatur på omkring 500oC, se Venus#Atmosphere_and_climate .

Från början hade Venus hav av vatten. Solens utstrålning ökar sakta och vattenångan i atmosfären gör att temperaturen ökar ytterligare genom växthuseffekten (se fråga 12668 ). Den succesivt ökande mängden vattenånga i atmosfären ger högre temperatur, vilket ger mer vattenånga osv, Venus får en accelererande växthuseffekt, se Runaway_greenhouse_effect#Venus .

Till sist försvinner haven. Vattenångan i atmosfären spjälkas av solens UV-strålning till väte och syre. Vätet är mycket lätt och har därför hög hastighet i den termiska rörelsen. Det betyder att vätet sliter sig loss från Venus gravitationsfält och försvinner. Syret försvinner genom att oxidera ämnena på ytan, och så småningom byts växthusgasen vatten ut mot koldioxid från Venus inre.



/Peter E

Nyckelord: växthuseffekten [36]; Venus [11]; Mars [12];

1 http://csep10.phys.utk.edu/astr161/lect/venus/greenhouse.html
2 https://www.aip.org/history/climate/Venus.htm

*

Blandat [20391]

Fråga:
Vilka positiva och negativa saker finns det om jordens atmosfär?
/Amanda G, Kyrk, Stokholm

Svar:
Jordens atmosfär är helt avgörande för livet på jorden:

* Djur behöver syre och växter behöver koldioxid (fråga 1550 ).

* Utan atmosfär kan inte flytande vatten existera (jämför Mars, fråga 7646 ).

* Atmosfären skyddar från skadlig strålning från världsrymden och solen.

* Växthuseffekten gör att jordens medeltemperatur blir c:a +15 grader i stället för -24 grader (fråga 16846 ).

Eftersom atmosfären är så helt avgörande är det svårt att hitta något negativt.
/Peter E

Nyckelord: jordens atmosfär [12];

*

Ljud-Ljus-Vågor [20758]

Fråga:
Hej!

Hur omvandlas kortvågig uv-strålning till långvågig infraröd strålning när den träffar jordytan? Mörka ytor har lägre albedo än ljusa ytor, vad beror detta på?
/Christian H, Högsätra skola, Lidingö

Svar:
Ljus som faller in mot jordytan absorberas. Energin hos den absorberade strålningen förvandlas till värme (slumpmässig rörelse hos atomer/molekyler). Detta värmer upp ytan.

Alla kroppar med en temperatur överstigande absoluta nollpunkten strålar med en för temperaturen karakteristisk fördelning, Plancks strålningslag, se fråga 12397 (figuren nedan). För solytan, som är ungefär 6000 grader, är maximum för utstrålningen i synligt ljus. Jordytan är c:a 15oC och strålar därför i infrarött.

Jordens atmosfär är transparent för det från solen inkommande ljuset, men inte transparent för den från jordytan kommande infraröda strålningen. Det är detta som orsakar växthuseffekten, se fråga 12668 .

För reflektion se fråga 17168 .



/Peter E

Nyckelord: Plancks strålningslag [6]; temperaturstrålning [29];

*

Ljud-Ljus-Vågor [20940]

Fråga:
Jag undrar hur det kommer sig att jorden är en approximativ svartkropp? När jag söker på detta kommer det endast upp fakta om huruvida solen är en approximativ svartkropp. Jag vet att det beror på jordens effekt etc, men hur visar man detta m.h.a. beräkningar?
/Emelie L, Nacka Gymnasium, Saltsjö-Boo

Svar:
Wikipedia: En svartkropp är ett objekt som absorberar all infallande elektromagnetisk strålning, och därmed inte reflekterar någonting. Detta medför att den strålning som avges från kroppen inte kan bero på den strålningsmiljö som kroppen befinner sig i, utan enbart på inre egenskaper hos kroppen.

Det visar sig då att den enda egenskap som påverkar den avgivna svartkroppsstrålningen är svartkroppens absoluta temperatur. Det är inte praktiskt möjligt att tillverka ideala svartkroppar, men man kan komma ganska nära en sådan konstruktion i form av en låda som invändigt har svarta väggar och som någonstans har ett mycket litet hål. Den strålning som lämnar hålet är till en god approximation fördelad på olika frekvenser i enlighet med Plancks strålningslag. (Svart_kropp )

Plancks strålningslag beskrivs i fråga 12397 . Från denna kan man härleda Stefan–Boltzmanns lag (totala utstrålningen) och Wiens förskjutningslag (positionen av maximum hos planckfördelningen). Se fråga 12397 .

Nedanstående figur (från länk 1) visar planckstrålningen från solen och jorden. (Observera att solens kurva ligger mycket högre eftersom den har högre temperatur.)

Fördelningen för jorden är en idealisering. I verkligheten finns det naturligtvis bland annat reflekterat solljus om vi skulle betrakta jorden från rymden. Vi skulle även se absorptionsband i fördelningen. Även för solen orsakar atmosfären absorptionslinjer.

Om vi tillämpar Wiens förskjutninglag får vi
För solen: våglängd för maximum = 0.002898/5780 = 500 nm
För jorden: våglängd för maximum = 0.002898/290 = 10 mikrometer

Detta stämmer bra med värdena i figuren nedan.

Det är förskjutningen i strålningsenergi från infallande synligt ljus till mikrometerstrålning som orsakar växthuseffekten genom att utstrålningen från jorden fångas upp i atmosfären, framför allt av koldioxid, se fråga 12668



/Peter E

1 https://www.researchgate.net/figure/Black-body-emission-curves-from-the-sun-T-5780-K-and-the-earth-T-290-K-showing_fig1_325914607

*

Universum-Solen-Planeterna [20975]

Fråga:
Istider och Milankovitch cykler
/Veckans fråga

Ursprunglig fråga:
Hej! Istider kommer och går, och de verkar antas bero på jordens omloppsbanas kring solen form; ju ovalare desto kallare (delvis enligt Keplers lagar). När blir banans avlånghet istidsgivande härnäst? Blir istiden alls av med nuvarande uppvärmning?
/Thomas Å, Knivsta

Svar:
Thomas!

Det är inte så enkelt som det framställs i fråga 14214 . Dels finns det fler möjliga effekter som kan påverka temperaturen och dessa kan ha positiva och negativa återkopplingar, se Ice_age#Causes . Det är inte givet att excentriciteten är dominerande även om perioden - drygt 100000 år - stämmer väl med temperaturdata, se 100,000-year_problem .

Bilden nedan från Milankovitch_cycles visar variationen hos

jordaxelns lutning (blå)
jordbanans excentricitet (grön)
perihelium longituden (lila)
Precessionsindex (rödbrunt)
Solinstrålning vid 65 grader N latitud (svart>

De nedre kurvorna är geologiskt uppmätta temperaturdifferanser.

Dessa data kan anses mycket tillförlitiga då de kommer från en mycket omfattande parametricering av positioner och banelement av hela solsystemet, se VSOP_(planets) .

Det är klart att en stor temperaturhöjning pga växthuseffekten skulle kunna omöjliggöra nästa istid genom att iskalotterna inte kan bildas. Observera emellertid att vi talar om olika tidsskalor: 100000 år och några 100 år.

Se även Ice_age#Variations_in_Earth's_orbit_(Milankovitch_cycles) och fråga 830 .



/Peter E

Nyckelord: istider [8]; Milankovitch cykler [3]; växthuseffekten [36];

*

Blandat [21264]

Fråga:
Jag undrar om den mänskligt genererade växthuseffekten, det vill säga klimatförändringarna , i ett allra värsta-scenario kan få jorden så varm att det inte har någon betydelse att en pol alltid har en midvinterperiod? Det vill säga om värmen av klimatförändringarna någonsin kan bli så stark att den får midvinterkölden som finns på grund av att solen ej kan nå polen, att bli över noll grader?

Jag menar: kan man hoppas på att det aldrig kommer att kunna bli olidligt varmt på hela jorden samtidigt just tack vare att polerna ju alltid kommer att ha solfrånvända perioder?
/Tina W

Svar:
Om jorden blir mycket varmare än nu skulle mer av havets vatten förekomma i gasform i atmosfären. Eftersom vatten är en växthusgas höjs då temperaturen ytterligare och man får en accelererande växthuseffekt.

Detta tycks ha hänt med planeten Venus där temperaturen är c:a 500 grader Celsius, se fråga 20231 .

Se Runaway_greenhouse_effect för mer om den accelererande växthuseffekten.
/Peter E

Nyckelord: växthuseffekten [36];

*

Materiens innersta-Atomer-Kärnor [21314]

Fråga:
Är kärnkraft farligt?
/Veckans fråga

Ursprunglig fråga:
Hej! Är kärnkraft farligt? Jag har precis sett en föreläsning från John Hopkins Applied Physics Laboratory med Bret Kugelmass.

Föreläsaren hävdar att historiskt sett så har aldrig en uran/vattenreaktor skapat mänskliga katastrofer. Det är bara plutonium/grafitmodererade reaktorer som gjort det (Tjernobyl). I princip menar han att Harrisburg och Fukoshima inte fick så allvarliga konsekvenser. Stämmer detta?

Han menar att det är den hårda lagstiftningen som gjort att det blir en katastrof.

Utöver detta menar han att kärnavfall inte är så farligt egentligen. Enligt honom är Iodine-131 den farliga komponenten i avfallet och det blir ofarligt efter 2 månader. Stämmer det?

Han hävdar också att om hela världen använder uran så har vi energi i miljoner av år. Stämmer det?
/Magnus L

Svar:
Nej, generellt sett är användandet av kärnenergi inte farligt. Låt oss titta på de tre allvarligaste incidenterna du nämner:

Three Mile Island

Three Mile Island (TMI) är ett sedan september 2019 nedlagt kärnkraftverk, som ligger vid Susquehannafloden i Londonderry Township, Dauphin County, Pennsylvania i USA. Anläggningens första reaktor togs i drift 1974 och stängdes i september 2019. Reaktor nummer 2 som startades 1978 totalförstördes 1979 i en härdsmälta, i folkmun kallad Harrisburgolyckan. (Three_Mile_Island )

Reaktorn totalförstördes (härdsmälta) men reaktorinneslutningen höll och mycket lite radioaktivitet kom ut i omgivningen. I dag ser många denna olycka som en demonstration av att vår konstruktion av kärnkraftverk är mycket säker.

Se Three Mile Island

Tjernobyl

Tjernobylkatastrofen var en mycket allvarlig reaktorolycka i kärnkraftverket i Tjernobyl norr om Kiev i Ukraina (som då var en Sovjetrepublik). Olyckan inträffade natten till lördagen den 26 april 1986 klockan 01.23.45,[1] (lokal tid) när reaktor fyra i utkanten av staden Prypjat förstördes genom en explosion och ett moln med radioaktiva partiklar spreds med vindarna över stora delar av Europa. (Tjernobylkatastrofen )

Den värsta kärnkraftsolyckan vi haft. Man fick en vätgasexplosion, med det var att moderatorn var brännbar (grafit) och avsaknaden av en stadig reaktorinneslutning som gjorde att utsläppen av radioaktivitet blev så stora.

Se Tjernobyl

Fukushima

Fukushima-olyckan avser en serie haverier och utsläpp av radionuklider vid kärnkraftverket Fukushima I som följde jordbävningen vid Tohoku den 11 mars 2011.

Tre av verkets sex block var vid tillfället i drift och snabbstoppades, då jordbävningen slog ut det yttre elnätet. Den tsunami, som följde 56 minuter efter jordbävningen, slog ut de reservgeneratorer som användes för reaktorernas kylning. Endast batterikraft återstod då och ungefär 50 minuter senare upphörde nödkylsystemet att fungera i block 1 och 2 och efter ytterligare 1,5 dygn även i block 3. Därefter saknade såväl härdar som bränslebassänger kylning, vilket ledde till partiella härdsmältor med vätgasexplosioner och utsläpp av radioaktiva ämnen som följd. (Fukushima-olyckan )

De tre havererade blocken var av typen lättvattenkylda kokarreaktorer med anrikat uran som bränsle (se Fukushima_Daiichi_nuclear_disaster#Plant_description .

Se Fukushima

Fjärde generationens reaktor

Fjärde generationens reaktor (Gen IV) är en benämning för sex olika typer av kärnreaktordesign, som valts ut som särskilt lovande för framtida reaktorer. De är för närvarande föremål för intensiv forskning. Reaktorerna avses användas i kärnkraftverk för att som i dag främst ta tillvara elektrisk energi från kärnbränslen. (Fjärde generationens reaktor )

Nu till dina frågor.

Harrisburg var en fullständig härdsmälta men mycket lite radioaktivitet slapp ut, så olyckan hade liten påverkan på människor, undantaget en möjlig rädsla för utsläpp. Om man så vill kan man säga att haveriet visade att de vanliga vattenkylda reaktorerna är mycket säkra (undantaget ekonomiska konsekvenser).

Tjernobyl är en helt annan typ av reaktor som saknade inneslutning. Trots de allvarliga konsekvenserna (flera akut döda, sena cancerfall, ett stort område evakuerat) kan vi räkna bort denna (enligt Kugelmass definition) eftersom reaktortypen inte existerar utanför det gamla Sovjet-blocket.

Fukushima är en standardreaktor i västvärlden. Att inte klassa haveriets konsekvenser (Fukushima_Daiichi_nuclear_disaster#Aftermath ) som allvarliga är både oärligt och korkat! Nej, ingen människa dog av akuta strålskador, men sena cancerfall och skador pga evakuering kan inte försummas.

Hur lagstiftningen skulle orsaka katastrofer begriper jag inte, det måste nog utvecklas.

Jod-131 har visserligen kort halveringstid (8 dagar), men även cesium-137 (med halveringstid 30 år) är skadligt.

Vanliga reaktorer använder 0.7% av uranet i bränslet (uran-235). Det finns reaktorer som använder allt uran och som även kan köras med t.ex. thorium. Genom att använda dessa extra isotoper som bränsle är tillgången på bränsle i praktiken obegränsad.

Bret Kugelmass är en professionell kärnenergi-lobbyist, men min åsikt är att uttalanden som ovan med hårdvinklade påståenden är snarast negativa för kärnenergins framtid. Om vi vill stoppa ökningen av CO2 (global uppvärmning, växthuseffekten) är sol och vind basresurser, men kärnenergi behövs som ett komplement. Men då måste man ta säkerhetsfrågorna mycket mer på allvar genom att designa "idiotsäkra" reaktorer, se Fjärde generationens reaktor ovan.

Länk 1 innehåller en intervju av Kugelmass. Länk 2 är en sammanfattning på svenska om fjärde generationens reaktorer.
/Peter E

Nyckelord: Tjernobyl [12]; Fukushima [6]; Three Mile Island [3]; kärnenergi [19]; växthuseffekten [36];

1 https://medium.com/@SustainabilityExplored/nuclear-an-old-new-solution-interview-with-bret-kugelmass-f1f09cc179b6
2 https://energiforsk.se/media/27042/broschyr-fjarde-generationens-karnkraft.pdf

*

Sök efter    

Skriv de ord du vill söka på i sökfältet ovan och klicka på sökknappen. Uteslut ord genom att sätta - (minus) före ordet. Ordgrupper definieras med hjälp av "...". Sökningar är oberoende av stora och små bokstäver.

Exempel:

helium "kalle anka"
Sök på 'helium' och ordgruppen 'kalle anka'
orgelpipa
Sök på 'orgelpipa'
orgel -gitarr
Sök på 'orgel' men inte 'gitarr'

 


sök | söktips | Veckans fråga | alla 'Veckans fråga' | ämnen | dokumentation | ställ en fråga
till diskussionsfora

 

Creative Commons License

Denna sida från NRCF är licensierad under Creative Commons:
Erkännande-Ickekommersiell-Inga bearbetningar
.